Peds EOR
A 6-week-old term girl presents to the ED on a winter evening with a three-day history of increased nasal congestion, difficulty sleeping, and difficulty breathing. She has slightly decreased oral intake but is still producing plenty of wet and soiled diapers. She has no significant medical history, but she has two older siblings who have also had a mild cold in the last week. Her vital signs on admission are: T 38.3, HR 128, RR 67, BP 90/51, and her oxygen saturation on room air is 90%. On exam, the infant is fussy but consolable, and in mild respiratory distress with moderate intercostal retractions. On auscultation, decreased air movement with coarse rales are audible bilaterally, but no wheezing is noted. Which of the following is the most appropriate next step in management? A) Airway suctioning and supplemental oxygen B) Albuterol breathing treatment C) Chest X-ray D) Intravenous methylprednisolone 1 mg/kg
A) Airway suctioning and supplemental oxygen
A 12-year-old girl presents to the clinic with a two-day history of a rash and recent known exposure to poison ivy. The rash is erythematous with scattered vesicles localized to a 6-inch segment along the medial aspect of the lower leg. There is no drainage noted and the patient denies systemic symptoms such as fevers or chills. Which of the following is the most appropriate treatment option? A) Clobetasol propionate 0.05% ointment twice daily for 14 days B) Hydrocortisone 1% ointment twice daily for seven days C) Prednisone 1 mg/kg tablet once daily for six days D) Tacrolimus 0.03% ointment twice daily to affected area until symptoms resolve
A) Clobetasol propionate 0.05% ointment twice daily for 14 days -High potency steroid is the TOC for allergic contact dermatitis
A 6-year-old boy presents to his pediatrician for follow-up due to left ear pain and drainage. He was seen in the same office two days ago, diagnosed with acute otitis media, and prescribed a 10-day course of amoxicillin. Since starting the antibiotic, his parents have not noticed any significant changes until this morning when he suddenly cried out in pain, holding his left ear. There is no history of trauma. His ear pain has decreased since this morning, but his parents have noticed a clear, sticky fluid around the opening to his left ear. On examination, the patient does not complain of pain with pulling the outer ear or pushing on the pinna or tragus. Using the otoscope, clear fluid, but no blood, can be visualized in the external canal. The light reflex usually seen on the tympanic membrane is obscured, and a small perforation can be seen on the anterior-inferior quadrant. Weber and Rinne tests confirm conductive hearing loss in the left ear. Which of the following is the most appropriate next step in management? A) Continue antibiotic course with close observation B) Discontinue oral antibiotic treatment C) Otolaryngology consult for tympanostomy tube placement D) Switch to a topical antibiotic treatment
A) Continue antibiotic course with close observation
A 1-day-old term infant is seen at his mother's bedside in the postnatal ward for a newborn exam. His mother reports he vomited a few hours after the first feeding and has been fussy but consolable. She has not noticed any other problems. She reports a normal pregnancy. On examination, the infant has a mildly distended abdomen. An abdominal film is ordered and notable for two prominent air bubbles in the stomach and duodenum. Which of the following is the most likely diagnosis? A) Duodenal atresia B) Hirschsprung disease C) Jejunal atresia D) Pyloric stenosis
A) Duodenal atresia -congenital abdominal obstruction due to failure of the duodenum to recanalize in early fetal development
A 10-year-old boy presents to your office with complaints of dark, cola-colored urine that developed yesterday. His mother states that he had a sore throat about a week ago that resolved on its own. Upon physical examination, you notice some puffiness in the periorbital areas bilaterally. His blood pressure is within normal limits. What is the most likely underlying cause of his condition? A) Group A beta-hemolytic Streptococcus infection B) Henoch-Schönlein purpura C) Methicillin-resistant Staphylococcus aureus infection D) Systemic lupus erythematosus
A) Group A beta-hemolytic Streptococcus infection *MC infectious cause of acute glomerulonephritis*
A 3-year-old girl presents to the ED due to concern for right arm pain. Her parents report they were holding her hand while preparing to cross a street today and had to pull her back abruptly because she started to dart into oncoming traffic. She cried immediately and since then she has not been using her right arm at all. On your exam, the child is well-appearing, in no acute distress, quietly holding her right arm in a flexed position close to her body. There is no visible deformity of the affected arm, and on exam, no focal areas of swelling, tenderness, or bruising are noted; no other injuries are noted in other locations. Which of the following is the most appropriate next step in management? A) Manual reduction B) Orthopedic consult C) Reduction under sedation D) X-ray of the affected arm
A) Manual reduction (TOC for Nursemaid's elbow)
A 16-year-old girl who is a ballet dancer presents to your clinic for her annual checkup. She has been training four hours per day, seven days a week for the past five months for an upcoming national competition. She has lost 15 pounds and plans on losing some more to "look better". She has not had her menstrual cycle in the past four months. She is afebrile, with a heart rate 60 beats per minute, blood pressure 90/60 mm Hg, and respirations 11 breaths per minute. She is 5 ft. 2 in. tall and weighs 82 lbs. Her body mass index is 15 kg/m2. Physical examination is notable for dry mucous membranes, dental caries, and calluses on the dorsum of her right hand. She also has fine hair over her cheeks. Which of the following is the first priority in caring for this patient? A) Nutritional rehabilitation B) Pharmacotherapy C) Psychoeducation D) Transcranial magnetic stimulation
A) Nutritional rehabilitation
A four-year-old patient presents to the clinic with his mother, who complains that he is "wall-eyed". The child is demonstrating well-coordinated, well-aligned gaze during the exam. A cover test shows no abnormalities, but a cover-uncover test is positive for slight medial repositioning of the covered eye immediately upon uncoverage. Visual acuity is 20/20, red reflex is present bilaterally, and fundoscopic examination is normal. Which of the following is the best choice for ongoing care of this patient? A) Observation with regular follow-up with ophthalmologist every six months B) Observation with regular follow-up with primary care provider every six months C) Occlusion therapy and visual training exercises D) Surgical extraocular muscle resection
A) Observation with regular follow-up with ophthalmologist every six months -f/u with a PCP is not enough -Occlusion therapy, visual training exercises, and surgery are options but secondary
A 13-month-old girl presents to her pediatrician for a routine well-check. Other than a prior hospitalization for influenza-associated bronchiolitis, she has no significant medical history, no current illness, and is otherwise healthy. Because it is now autumn and influenza has been reported, a seasonal influenza vaccine is available and recommended. Her mother remembers her friend's children required more than one vaccination for seasonal influenza and asks if her daughter will also need a second vaccination. Which of the following is the most appropriate advice? A) A second vaccination is only recommended if using certain vaccine formulations B) A second vaccination is recommended C) If a child contracted influenza in the previous season, a second vaccination is not needed D) Only a single dose is required
B) A second vaccination is recommended
A newborn examination is conducted on a 1-day-old, 36-week gestation infant born via spontaneous vaginal delivery with no complications. His physical exam is unremarkable except for an empty left scrotum. On closer examination of his genitalia, one hand is used to gently express any palpable tissue from the anterior superior iliac spine along the inguinal canal towards the scrotum, which is held by the examiner's other hand. During this maneuver, the hand on the scrotum receives testicular tissue which is gently held in the scrotum for at least one minute. Upon letting go, the testis disappears out of the scrotum. Which of the following is the most likely explanation for these findings? A) Absent testis B) Ectopic testis C) Retractile testis D) True undescended testis
B) Ectopic testis -testis can be palpated and pushed into the scrotum but will "spring" back out without pressure
An 11-month-old girl is brought to the ED by her mother. Mom reports that when she picked her up from daycare in the afternoon the infant seemed fine, but she didn't eat much for dinner and subsequently vomited. There were what looked like clumps of blood in her vomit. She has had no recent illnesses and no known injuries. Her vital signs are normal on presentation. On exam, she is a well-appearing, playful child. Her physical exam, including abdominal exam, is generally unremarkable. However, there appears to be dried blood in her left nare. No foreign bodies or masses are visualized. What further history is most important to obtain? A) Dietary inventory B) Events while at daycare C) Family history of bleeding disorders D) Gastrointestinal history
B) Events while at daycare
A 7-month-old boy presents to clinic after recent adoption from a Romanian orphanage. He has been having bulky stools several times each day. He also has a persistent cough and his parents were told that he was once treated for pneumonia. Additional medical history is unavailable. His weight-for-height is less than the 5th percentile, but he eats very well. Examination is remarkable for a thin child with temporal wasting, wet-sounding cough, scattered pulmonary rales, greasy stool in the diaper, and trace pedal edema. Which of the following tests is most likely to be positive in this patient? A) Echocardiogram B) Fecal elastase C) Karyotype analysis D) Stool ova and parasites
B) Fecal elastase -supporting test for cystic fibrosis -DOC is sweat chloride test
An 18-month-old adopted boy presents to the clinic for consult. He just recently transferred residence from another state where he used to have a pediatrician for his regular well baby consultation visits. The adoptive mother states that according to his previous physician, the boy has a ventricular septal defect and is need of early childhood intervention for developmental delay. Family histories of both biological parents are unknown. Upon physical examination, the child is both stunted and wasted with z scores less than 5% for both weight and height. Other pertinent findings include short palpebral fissures, a short nose, and a thin upper lip with a smooth philtrum. What is this child's most likely condition? A) Ehlers-Danlos syndrome B) Fetal alcohol syndrome C) Holt-Oram syndrome D) Turner syndrome
B) Fetal alcohol syndrome
A 9-year-old boy presents to his pediatrician with a burn injury sustained today when a hot beverage spilled onto both his thighs. His parents attempted to treat him with cool washcloths, but it took some time to remove his clothing initially. Now, there are blisters arising in the injured area. On assessment, the patient has two moist, weeping, erythematous areas across his anterior thighs with multiple small, intact, liquid-filled blisters, but he is otherwise well-appearing. He reports 7 out of 10 pain whenever he has to move, when the cool washcloths are removed, or when anyone or anything touches the affected areas. On exam, the area does blanch with pressure, and is exquisitely painful even with light touch. An estimated 4% of his body surface area is covered by the burn, his genitals are not affected, and there are no other injuries noted. Which is the most appropriate next step in management? A) Debridement of blisters and dressing of affected area with topical antibiotic B) Gentle cleaning and dressing of affected area with a moisturizing cream C) Immediate referral to the nearest ED D) Immediate transfer to the nearest burn center
B) Gentle cleaning and dressing of affected area with a moisturizing cream (TOC for superficial partial-thickness burn, blisters should not be burst if still intact)
A 12-year-old girl presents to her pediatrician for follow-up after a school screening was concerning for scoliosis. She has no significant past medical history, but her family history is notable for an older brother who was diagnosed with adolescent idiopathic scoliosis that did not require bracing or surgical correction. She denies injury, pain, neurologic changes, or changes to her bowel and bladder movements. Which of the following can be used to assess whether this patient's scoliosis is likely to worsen? A) Birth history B) Growth trajectory and signs of puberty C) Recent medical history and infectious diseases D) Respiratory history
B) Growth trajectory and signs of puberty (scoliosis is at the greatest risk for worsening during growth spurts)
An 18-month-old girl presents to the emergency department with respiratory distress. She was well until yesterday when she developed a mild cough and nasal congestion. Today, her cough became increasingly harsh, and her voice became raspy. On observation in her mother's arms, she is tachypneic, has moderate subcostal retractions, and makes an monophonic inspiratory noise with each breath. What medication should be immediately administered? A) Nebulized albuterol B) Nebulized racemic epinephrine C) Oral dexamethasone D) Vancomycin and ceftriaxone
B) Nebulized racemic epinephrine (+Dexamethasone) -epinephrine is necessary for immediately respiratory relief
An 18-month-old uncircumcised boy with no significant past medical history presents to the ED. His parents report increasing fussiness throughout the day and he seemed to be itching or playing with his diaper region more than usual. While changing his diaper this evening they noticed significant swelling of his foreskin and he vigorously fought having his diaper area cleaned. He is still producing wet diapers and has been eating fine. He has otherwise been healthy with no fevers or recent illnesses. On physical exam, the patient has notable swelling of the distal foreskin and a band of tissue proximal to the head of the penis appears constricting. Both the glans penis and the distal foreskin are swollen and exquisitely tender to manipulation. The penile shaft appears flaccid and normal. Which of the following is the most likely diagnosis? A) Balanoposthitis B) Paraphimosis C) Phimosis D) Tourniquet syndrome
B) Paraphimosis
A 4-year-old girl with no significant medical history presents to her pediatrician for a routine well-check. During the visit, her mother asks about a skin lesion she noticed recently. The area in question is a pale, firm, hyperkeratotic papule about 4 millimeters in diameter on the lateral aspect of the patient's right knee. When scraped gently with a tongue depressor, some of the overlying debris can be removed, revealing scattered, pinpoint, deep-red dots. The area is not painful. Which of the following is the most appropriate therapy? A) Cryotherapy with liquid nitrogen B) Reassurance and watchful waiting C) Shave biopsy with pathology consultation D) Topical salicylic acid
B) Reassurance and watchful waiting -MC caused by HPV -Most resolve spontaneously -Other options are treatment but painful so waiting is better
A 5-year-old boy with a recent history of constipation is seen by a pediatrician for a chief complaint of stool leaking onto his underwear. He was toilet-trained when he was three, but since then has had occasional periods of constipation which his mother has been treating with apple juice mixed with polyethylene glycol as needed. She believes the treatment works, but sometimes she doesn't know when to start giving it to the patient. Since he is more independent on the toilet now, she didn't realize he was having problems but noticed he had soiled his underwear and is concerned there is a new problem. Per the patient, he did not have a bowel movement today or yesterday and he cannot recall when the last bowel movement was. He does report the last time he had a stool it was painful. Rectal examination confirms the presence of hard, retained stool. Which of the following is the most appropriate next step in management? A) Abdominal X-ray and referral to gastrointestinal specialist B) Administer a sodium phosphate enema C) Begin polyethylene glycol and continue for at least six months D) Begin polyethylene glycol and continue until the problem resolves
C) Begin polyethylene glycol and continue for at least six months
A 6-month-old boy is brought to the emergency department by his frantic mother due to a generalized tonic-clonic seizure. Per the mother, her son's seizure lasted for a total of about five minutes. He is otherwise healthy, with no prior history of seizures, and all his immunizations are up to date. On physical examination, he is easily consolable, drinking milk from a bottle with a good suck. He moves all extremities vigorously and there are no focal neurologic deficits. His temperature is 103.5°F. If the patient were to have a recurrence of febrile seizure in the future, what medication can you prescribe that the parents may give at home? A) Adrenocorticotropin hormone B) Carbamazepine C) Diazepam D) Ethosuximide
C) Diazepam (TOC for a febrile seizure lasting more than 5 minutes)
A 12-year-old girl with no significant medical history presents to her pediatrician complaining of two weeks of joint pain. Initially, the pain started at the same time as a fever, first in her knees, then her ankles, and now she feels like her elbows are bothering her as well. Ibuprofen helps a great deal, but the pain is still present after the medication wears off. Her only recent illness was strep throat which was treated with amoxicillin. On physical exam, mild erythema and swelling around her elbows is appreciated and her lower extremities appear normal but are painful to move. The rest of her physical exam is also normal, including ear, nose and throat, cardiac, and skin exams. Which of the following is the most appropriate diagnostic test? A) Antistreptolysin O titer B) Complete blood count with differential C) Echocardiogram D) X-ray of affected joints
C) Echocardiogram -clinical presentation concerning for acute rheumatic fever Jones major diagnosis criteria: arthritis, carditis, Sydenham chorea, erythema marginatum, and subcutaneous nodules
A 3-year-old boy is referred to the pediatric hematologist with a chief complaint of easy bruising, nosebleeds and decreased activity for one week. He has no history of fever or appetite changes. His past medical history is remarkable for multiple congenital anomalies but no workup has been done. There is no travel history, history of recent illnesses, or known exposure to toxins. The physical examination reveals an active boy with short stature, notable hyperpigmentation of the trunk, neck, and intertriginous areas, as well as café-au-lait spots and vitiligo. The boy also has microcephaly, broad nasal base, epicanthal folds, micrognathia, anicteric sclera, pale palpebral conjunctivae, no cervical lymphadenopathy, clear breath sounds, adynamic precordium, no heart murmur, soft non-tender abdomen, full equal pulses, and capillary return time of less than two seconds. Laboratory tests reveal hemoglobin 7.9 g/dL, hematocrit 24%, platelet count 12,000/mcL, WBC 3,000/mcL with 90% lymphocytes, absolute neutrophil count 210 cells/L, and reticulocyte count 0.5%. Which of the following is the most likely diagnosis? A) Beta thalassemia B) Diamond-Blackfan anemia C) Fanconi anemia D) Paroxysmal nocturnal hemoglobinuria
C) Fanconi anemia -autosomal recessive -pancytopenia and congenital anomalies -TOC: supportive care (transfusions, antibiotics, corticosteroids, androgen therapy), stem cell transplant
A 5-year-old boy presents to the emergency department because of difficulty breathing that began a few hours prior to consult. The caretaker reports that he was apparently well until he developed a fever and sore throat yesterday. He has no cough. On examination he is noted to be flushed, maintaining a sitting position with his neck hyperextended and arms braced forward. He is also febrile and has low-pitched inspiratory stridor. Which of the following was the most common etiologic agent responsible for the boy's disease in the pre-vaccination era? A) Candida sp B) Group A Streptococcus C) Haemophilus influenzae type b D) Staphylococcus aureus
C) Haemophilus influenzae type b *MC cause of acute epiglottitis*
You partner with a local school to create a checklist of symptoms that the teachers fill out to aid in the diagnosis of attention deficit hyperactivity disorder. Which combination of symptoms would be most consistent with a diagnosis of attention deficit hyperactivity disorder? A) Actively defies or refuses to follow teachers' rules, is easily annoyed by classmates and is frequently angry or resentful, argues with teachers B) Bullies or intimidates peers, leaves seat when remaining seated is expected, lies to get out of trouble, skips school without permission C) Has difficulty organizing tasks and activities, blurts out answers before questions have been completed, has difficulty waiting for his or her turn, is forgetful in daily activities D) Is easily distracted, frequently worries, is afraid to go to school or be called on by the teacher to answer questions
C) Has difficulty organizing tasks and activities, blurts out answers before questions have been completed, has difficulty waiting for his or her turn, is forgetful in daily activities
A 12-month-old boy presents to your clinic with his concerned mother for abnormal movements. The mother reports that the child has sudden flexion and stiffening of the neck, limbs, and trunk followed by gradual extension and relaxation. These jerking spells have become more frequent and occur in clusters throughout the day. The baby will cry briefly after the spells and then become sleepy. The mother also reports that about a month ago, he started walking consistently; however, he is no longer doing that at present. Which electroencephalographic finding is most consistent with the diagnosis? A) Abnormally slow background with diffuse slow spike and slow wave (< 2.5 Hz) activity B) Frequent spike and wave discharges in the centrotemporal region C) Hypsarrhythmia D) Normal background for age with 3-Hz generalized spike and wave discharges
C) Hypsarrhythmia -typical EEG presentation of infantile spasms -pt's clinical presentation most likely matched to infantile spasms West syndrome: triad of infantile spasms, developmental regression, and hypsarrhythmia on EEG
A 2-year-old boy presents to the clinic with his mother for a facial rash. His mother says the rash began one day ago as two small bumps and has gradually spread around his nose and mouth region. The child frequently scratches the area and she has noticed drainage. The patient has otherwise been acting his usual self and has had no fevers, nasal drainage, or cough. Physical exam reveals several papules and erythematous vesicles with overlying honey-colored crust surrounding his nose and mouth. Which of the following is the most likely diagnosis? A) Contact dermatitis B) Herpes labialis C) Impetigo D) Varicella infection
C) Impetigo
A 17-year-old teenager presents to your clinic office for a new visit after he was referred by the school psychologist for depressed mood. The patient reports that for the past three to four years he has been having a persistently low mood. More recently, he has been having worsening symptoms of depression due to being rejected by his dream college and the recent divorce of his parents which disrupted the home routine. He has very few friends and he feels that he has no one to turn to for help or support. He feels very hopeless and that "life is not worth living anymore." The patient reports that he is willing to take medication to help him feel better. There are no guns at home, but he says they have a medicine cabinet filled with prescription medications from his other family members at home. He reports that he wonders if overdosing with medication would be easier than using a knife or firearm. Which of the following is the next best step? A) Develop a safety plan with the patient and parent, and follow up the patient closely within the next few days B) Have the patient sign a contract for safety and refer to psychiatry C) Inform the patient that you will send him to the emergency department for evaluation D) Start selective serotonin reuptake inhibitor medication immediately for his depression
C) Inform the patient that you will send him to the emergency department for evaluation *high concern about the patient being a threat to himself*
A 2-year-old girl presents with her grandmother to clinic due to constipation. The grandmother observes that since birth and despite recurrent use of stool softeners, the child only moves her bowels once a week. There is no fecal soiling or diarrhea noted. She was born full term and her mother had an unremarkable prenatal history. Her grandmother states that the child was not discharged immediately after birth since she did not pass any stool for 48 hours after birth. However, she was well thereafter and has not been in the hospital since. Which of the following would best explain the girl's symptoms? A) Failure of the bowel lumen to recanalize B) Incomplete obliteration of the omphalomesenteric duct C) Lack of ganglion cells in the submucosal and myenteric plexus D) Nonrotation of the bowel in and out of the abdominal cavity
C) Lack of ganglion cells in the submucosal and myenteric plexus (pathophysiology of congenital aganglionic megacolon, or Hirschsprung disease)
2-month-old girl born at 34 weeks gestation presents with her parents for a routine well-check and vaccinations. She had no significant adverse reactions to her previous hepatitis B vaccination. Her mother was positive for group B Streptococcus during pregnancy but was adequately treated during delivery. The infant has no other significant medical history and is fussy but consolable on presentation. Her parents report she started having acute watery diarrhea today and hasn't been feeding well, however, she is afebrile. They are hesitant to start her immunizations for a number of reasons, including the fact that her 3-year-old brother is currently being evaluated for autism. Which of the following is a valid reason to delay her 2-month immunizations? A) History of group B Streptococcus exposure B) History of premature birth C) Moderate acute illness without fever D) Sibling with autism
C) Moderate acute illness without fever CIs for vaccines: hx of severe allergic reaction, known or fam hx of immunodeficiency Precautions for vaccines: moderate or severe acute illness with or without fever, recent administration of antibody-containing blood product, hx of thrombocytopenia, progressive neurologic disorder
A 4-year-old boy presents to urgent care with a two-day history of crampy abdominal pain and non-bloody, non-bilious vomiting and diarrhea. He had two episodes of large-volume watery diarrhea yesterday and today another two smaller episodes. He has felt nauseous or has vomited every time he has tried to eat or drink. His parents report in the last 24 hours he has become more listless and is now reluctant to eat and drink because he is afraid of vomiting. His vital signs on admission are: T 38℃, HR 110, RR 35, BP 91/50, oxygen saturation on room air 97%. On exam, the patient is awake and cooperative but not playful. His mucous membranes are mildly dry, he has hyperactive bowel sounds and mild tenderness to deep palpation throughout his abdomen, but no rebound or guarding. His extremities are cool with a capillary refill of about three seconds. Which of the following is the most likely explanation for these findings? A) Hypovolemic shock B) Minimal dehydration C) Moderate dehydration D) Severe dehydration
C) Moderate dehydration
A 6-month-old infant presents to the emergency department with his mother for lethargy. She reports that the baby has been unusually sleepy and feeding poorly over the past few days. Today, the mother noticed that she had a very weak cry and that she hardly ate anything. The mother suggests that the baby may have caught a cold that has been going around the house. Birth history and medical history are unremarkable. The infant missed her most recent six month well child visit with the pediatrician but she reports that the baby has been developing normally. When the mother is at work, her boyfriend takes care of the baby at home. On examination, the child is lethargic with vital signs within normal limits for age. You note a bluish gray patch on the child's right lower extremity and buttocks, as well as a 2 cm circular burn on the left arm. Which of the following is the most appropriate next step in the evaluation of this infant? A) Call Child Protective Services and report suspicion of child abuse B) Interview the mother's boyfriend alone C) Order computed tomography scan of the brain D) Order skeletal survey for suspicion of child abuse
C) Order computed tomography scan of the brain *any of the other answers are correct, but the with a high suspicion for non-accidental trauma or child abuse, an emergent injury needs to be ruled out first*
A five-week-old boy presents to the emergency department due to a ten day history of vomiting that has increased in frequency and forcefulness. The vomitus is described as previously ingested milk and is not bile-stained. The boy has a good appetite and looks well in between episodes; however, he is noted to be losing weight. Physical examination reveals a calm baby boy with anicteric sclerae, pink conjunctivae, clear breath sounds, adynamic precordium, no heart murmur, a soft non-tender abdomen with a firm, movable, olive shaped, mass palpated above and to the right of the umbilicus in the mid-epigastrium beneath the liver edge, full and equal pulses, and capillary return time of less than two seconds. Which of the following is the most likely diagnosis? A) Duodenal atresia B) Gastroesophageal reflux disease C) Malrotation D) Pyloric stenosis
D) Pyloric stenosis
A 4-week-old girl presents to an ophthalmologist due to concern about "cloudy" eyes. She is the first baby born to Indonesian parents who immigrated to the United States just two weeks ago. Her parents report she has otherwise been healthy. Her parents are also in good health. Mother reports a healthy pregnancy, though she had a flu-like illness while pregnant and did take acetaminophen for symptomatic relief. She had an uneventful vaginal delivery, although the infant was noted to be small for gestational age. The infant's clinical findings are compatible with congenital rubella infection. What is the most concerning risk factor for this condition? A) Infant small for gestational age B) Maternal acetaminophen use C) Maternal flu-like illness during pregnancy D) Recent emigration from a developing country
D) Recent emigration from a developing country (lack of rubella immunization programs)
A 7-year-old girl presents to the clinic complaining of intermittent fever and generalized skin lesions. The fever started one to two days before the appearance of skin lesions, which began as tiny papules and progressed rapidly to clear vesicles followed by pustules and scabs. These lesions are widely distributed, but are noted predominantly on the torso. The child's mother does not believe in the benefits of immunization therefore the girl has not received any vaccinations. One of the girl's classmates also had the same manifestations two weeks ago and has been absent from class since then. Which of the following is the most likely etiologic agent? A) Poison ivy B) Solenopsis invicta C) Staphylococcus aureus D) Varicella zoster virus
D) Varicella zoster virus
A 10-year-old boy presents with a five-day history of severe sore throat, right-sided otalgia, fever, and difficulty opening his mouth. He is speaking with a "hot potato" voice. Physical exam reveals his ears to be normal. Oropharyngeal exam shows bulging of the right soft palate with uvular deviation to the left. What is the most likely diagnosis? A) Epiglottitis B) Mononucleosis C) Peritonsillar abscess D) Streptococcal tonsillitis
C) Peritonsillar abscess
EPSDT Measurements 9 mos
Peds Tool
Refractometer should be done at
2, 3, 4, 5, then use eye chart prn
Breast Tanner Stage 3
Further breast enlargement and elevation of both breast and areola
Male Genitalia Tanner Stage 5
Genitalia of adult size and proportion
Pubic Hair Tanner Stage 2
Growth of sparse straight hair, primarily at the base of the penis or along the labia
Skills of Cognitive/Problem Solving Development
Intellect Thinking Remembering Categorizing Problem-solving Analysis/synthesis
EPSDT Measurements 18 and 24 mos
MCHAT screening form
Who Assesses Gross Motor Development
PCP PT
Male Genitalia Tanner Stage 1
Prepubertal genitalia
Functions Affected by Adult Loss of Speech/Language Development
Speaking Hearing
Innocent Pediatric Murmurs
Still's: MLSB, LLSB (6-8yrs) Pulmonary Flow: LUSB, Back (NB, 3-6 month resolution) Pulmonary Ejection: LUSB (8-14 yrs) Carotid Bruit Venous Hum: altered with turned head
What is the pathophysiologic mechanism of hyaline membrane disease?
Surfactant deficiency Hyaline membrane disease (Resp distress syndrome) results from alveoli collapse due to lack of adequate lung surfactant and immature lungs
A 17-year-old male presents to your office for evaluation. In addition to struggling academically, you notice he is tall for his age, has small testes, and gynecomastia. Upon further workup, what genetic finding will this patient most likely have?
47, XXY This patient's clinical findings are most consistent with Klinefelter syndrome. This is a genetic disorder in which patients have an extra X-chromosome and a karyotype of 47, XXY. Klinefelter syndrome is caused by the presence of an additional X-chromosome. Affected individuals are typically asymptomatic throughout childhood and present in adolescence or early adulthood. The typical clinical presentation includes tall stature, small testes, and gynecomastia. Further workup reveals a 47, XXY karyotype and typically, azoospermia. Almost all men with Klinefelter syndrome are infertile. Late onset of puberty should prompt further evaluation, however often at diagnosis, the window for any therapeutic interventions has passed.
A 4 year-old patient presents with episodic wheezing and a non-productive cough for the last 4 weeks. His symptoms are worse at night. Past medical history reveals a history of atopic dermatitis. Physical examination at this time is unremarkable. What is the most likely diagnosis?
Asthma Asthma is a chronic inflammatory disorder of the airways. It is characterized by episodic or chronic symptoms of airflow obstruction, breathlessness, cough, wheezing, and chest tightness. The strongest identifiable predisposing factor for the development of asthma is atopy.
A 3 year-old girl is diagnosed with atopic dermatitis. What disorders is this child at risk for in the future?
Asthma and allergic rhinitis Up to 50% of patients with atopic dermatitis develop asthma and/or allergic rhinitis in the future.
Murmurs heard best over the LUSB
Atrial Septal Defect Pulmonary Valve Stenosis Patent Ductus Arteriosus Cortication of the aorta Tetralogy of Fallot
A 5-year-old girl with a significant past medical history of eczema presents to the outpatient office with her mother complaining of numerous small, circular, flesh-colored umbilicated lesions that are located on the trunk and extremities. There are no lesions on the palms, soles or mucous membranes. They do not itch and do not appear to be filled with fluid. The lesions were first noticed about one month ago. What is the most likely diagnosis? A) Keratoacanthoma B) Molluscum contagiosum C) Pityriasis rosea D) Varicella
B) Molluscum contagiosum
A 4 week-old female is seen for a well child visit. Birth history is significant for breech presentation and weighing 9 pounds 8 ounces at delivery. What is indicated to rule out developmental dysplasia of the hip?
Physical examination A careful physical examination at birth and repeated evaluation at each well visit until the child walks is indicated to identify DDH. A high degree of suspicion is necessary in children with risk factors for DDH, such as positive family history, ligamentous laxity, breech presentation, female gender, large fetal size, and first-born status.
An 8 year-old boy is brought to a health care provider complaining of dyspnea and fatigue. On physical examination, a continuous machinery murmur is heard best in the second left intercostal space and is widely transmitted over the precordium. The most likely diagnosis is
patent ductus arteriosus Patent ductus arteriosus is classically described in children as a continuous machinery-type murmur that is widely transmitted across the precordium
An 8-year-old boy is brought to a physician because of palpitation, fatigue, and dyspnea. On examination, a continuous machinery murmur is heard best in the second left intercostal space and is widely transmitted over the precordium. The most likely diagnosis is
patent ductus arteriosus. Patent ductus arteriosus is classically described in children as a continuous machinery-type murmur that is widely transmitted across the precordium.
A mother brings in her 3 month-old infant and states that she has noticed a rash on her infant's scalp. Physical examination reveals the presence of erythematous and scaling crusty lesions involving the vertex of the scalp. What is the most appropriate initial intervention?
warm olive oil compresses This infant most likely has scalp seborrheic dermatitis ("cradle cap"). Initial treatment consists of warm olive oil compresses to remove any crusts followed by use of baby shampoo or mild hydrocortisone cream.
Language at 3 Years
1000 words 3-4 word phrases Strangers can understand 75% of language Tell stories
By what age should the posterior fontanel be closed?
2 months The posterior fontanel is typically closed at birth, but if open should close by 2 months of age
A 17-year-old man presents to the clinic complaining of hair loss. He states, "My father has a full head of hair. Why am I losing mine?" On exam, hair follicles of varying lengths and thicknesses are noted. Hair loss is noted at the vertex of the scalp, revealing healthy-looking underlying skin. The patient denies use of medications or supplements, pruritus, flaking, burning or bleeding scalp. Hair pull test results are negative. Which of the following is the most likely diagnosis? A) Alopecia areata B) Androgenetic alopecia C) Telogen effluvium D) Tinea capitis
B) Androgenetic alopecia (affected by genetics and androgens, aka men can have this even if their fathers do not)
A 4-month-old girl presents with two weeks of cough that includes severe coughing episodes with associated cyanosis, respiratory distress, and post-tussive emesis. Her mother and father both have upper respiratory infections. Which of the following is the treatment of choice? A) Amoxicillin B) Azithromycin C) Nebulized albuterol D) Reassurance
B) Azithromycin (TOC for perussis/whooping cough)
Which of the following laboratory abnormalities is most commonly noted in bulimia nervosa? A) Glycosuria B) Hypokalemia C) Metabolic acidosis D) Hyperalbuminemia
B) Hypokalemia Episodes of binge eating are followed by purging in the bulimic patient. Vomiting and laxative abuse are the most common methods of purging, leading to hypokalemia.
Which of the following primitive reflexes should begin to disappear at about 2-3 months of age in a normal infant? A) Moro B) Rooting C) Tonic neck D) Parachute
B) Rooting A newborn infant will turn its head toward anything that strokes its cheek or mouth, searching for the object by moving its head in steadily decreasing arcs until the object is found. The rooting reflex starts to disappear at about 2-3 months of age.
A 12-month-old girl, accompanied by her mother, presents to the clinic with a rash. The patient has a three-day history of fever ranging from 102-103°F with associated nasal congestion and diarrhea. The rash developed 24 hours after fevers had resolved, beginning around her neck and torso and gradually spreading to her arms and legs. Physical exam reveals a blanching maculopapular rash on the trunk and extremities. Which of the following is the most likely diagnosis? A) Erythema infectiosum B) Roseola infantum C) Rubeola D) Scarlet fever
B) Roseola infantum (rash developed as symptoms improved)
A 5 year-old is being evaluated for autism. Of the following, the finding most consistent with this diagnosis is A) extremely high IQ B) avoids eye contact C) easily consoled by hugging when hurt D) frequently plays "house" or "cops and robbers" with other children
B) avoids eye contact. Children with autism do not tend to make eye contact, and even avoid it.
A 3-year-old male presents to the emergency department with difficulty breathing. On physical examination, the patient is febrile and tachypneic. The patient's lateral neck radiograph is shown here. Which of the following signs on physical examination is most specific for this patient's disease? A) Cough B) Stridor C) Drooling D) Rhinorrhea E) Conjunctivitis
C) Drooling This patient's presentation is consistent with epiglottitis. Among the choices, drooling is the physical finding most specific for epiglottitis. Epiglottitis is a serious and rapidly progressive infection of the epiglottis and contiguous structures that affects children from age 3-7. It presents with fever, tachypnea, stridor, drooling, the "tripod position," and the "thumbprint sign" on lateral neck radiograph. It is classically caused by H. influenzae type B (Hib) but S. aureus and Streptococcus are now common causative agents due to the Hib vaccine. Epiglottitis is is a true airway emergency and requires immediate intubation in the OR. Patients remain intubated while receiving IV antibiotics for up to 72 hours. It must be differentiated from croup, which presents similarly and is not immediately life-threatening; croup shows the "steeple sign" on CXR and presents with cough and a lack of drooling.
A 9-year-old girl presents to the clinic for evaluation. Her mother is concerned she may have had a seizure yesterday and has since been too weak to play outside the house. On physical examination, the girl is found to be below the 5th percentile in height for age with normal vital signs. She has multiple dental caries, widening of costochondral junctions, bowing of the legs, and enlarged wrists and ankles. Which of the following laboratory screening studies suggest nutritional vitamin D deficiency? A) Hyperphosphatemia, hypocalcemia, and elevated parathyroid hormone B) Hyperphosphatemia, normal serum calcium, and elevated parathyroid hormone C) Hypophosphatemia, hypocalcemia, and elevated parathyroid hormone D) Hypophosphatemia, normal serum calcium, and low parathyroid hormone
C) Hypophosphatemia, hypocalcemia, and elevated parathyroid hormone
A 5-year-old girl presents with persistent rhinorrhea and chronic mild cough. On examination, she has clear nasal discharge, a bluish hue to her nasal mucosa, a crease across her nasal bridge, and clear secretions in the posterior pharynx. Which of the following medications is most appropriate to treat her condition? A) Amoxicillin-clavulanate B) Diphenhydramine C) Intranasal glucocorticoid D) Nasal decongestant
C) Intranasal glucocorticoid (TOC for allergic rhinitis)
Which of the following is caused by a persistence of the omphalomesenteric duct beyond fetal development? A) Congenital diaphragmatic hernia B) Duodenal atresia C) Meckel diverticulum D) Omphalocele
C) Meckel diverticulum
A 9 year-old presents with increasing shortness of breath while playing basketball recently. On examination, radial pulses are exaggerated while femoral pulsations are weak. Chest radiograph shows rib notching and a mildly enlarged heart. What is the most likely diagnosis for this patient?
Coarctation of the aorta In older children, the ECG and chest x-ray usually show left ventricular hypertrophy and a mildly enlargedheart. Rib notching may also be seen in older children (>8 years old) with large collaterals.
Skills of Speech/Language Development
Communication Words and symbols (semantics) Grammar (syntax) Pragmatics Articulation/voice/fluency
A 36-month-old boy is being evaluated at your office. Parents are concerned because he is short and has evidence of developmental delay with hypotonia. He was born at term with a birth weight of 3250 grams. He has a history of poor suck that required physical therapy. Recent fluorescence in situ hybridization was positive for a deletion on chromosome 15. This patient is at a higher risk of suffering from which of the following conditions? A) Alzheimer disease B) Congenital heart disease C) Hypocalcemic seizures D) Obesity
D) Obesity -Prader-Willi syndrome is the MC syndromic form of obesity Prader-Willi syndrome S/S: neonatal hypotonia, learning difficulties, food-seeking behaviors, decreased ability to vomit, increased tolerance of pain
6 Week EPSDT
F/u newborn screen results Review Tylenol dosing and fever precautions Review infant sleep patterns Review development (social smile, follows with eyes, lifts head during tummy time) Review GER precautions vs normal spit up Review recommended vaccines Schedule 4 month EPSDT
A 20 year-old female presents with episodes of binge eating, overuse of laxatives, and periods of starvation. What is the best treatment option for this patient?
Fluoxetine (Prozac) Fluoxetine, a SSRI, is the drug of choice for the treatment of bulimia nervosa.
What scabicide has been associated with neurotoxicity in infants and young children?
Lindane (Kwell) Lindane (Kwell) is concentrated in the CNS and toxicity from systemic absorption in infants has been reported.
Who Assesses Social/Emotional Development
PCP Psychologist Mental health therapist
Who Assesses Cognitive/Problem Solving Development
PCP Psychologist Optometrist or ophthalmologist
Who Assesses Speech/Language Development
PCP SLP Audiologist
What is the most effective prophylaxis against respiratory syncytial virus (RSV) infection in the general pediatric population?
Proper hand-washing techniques Proper hand washing and reduction in exposure is most effective in general population to prevent RSV.
Skills of Fine Motor Development
Tool use Dexterity Precision Bi-manual coordination
Functions Affected by Adult Loss of Gross Motor Development
Walking
Spina bifida occulta is usually detected by which initial diagnostic evaluation?
X-ray of the spine X-ray or MRI is the definitive test to diagnose spina bifida occulta, showing the vertebral bony defect.
The birth weight of an infant has usually tripled by
one year A child triples his/her birthweight by one year.
Breast Tanner Stage 2
"Breast bud" forms below the areola
Functions Affected by Adult Loss of Cognitive/Problem Development
"Mental functions" -adaptive functioning -memory -problem-solving, goal-setting, judgment
Number of Words at 18-24 Months
10-20 words
According to the Advisory Committee on Immunization Practices, what is the recommended age range for the first or initial MMR (measles, mumps and rubella) vaccination?
12-15 months The recommended age range for the initial vaccination against MMR is 12-15 months.
Number of Words at 12-15 Months
5-10 words
In infants, the eyes should move in parallel without deviation by the age of
6 months Intermittent alternating convergent strabismus is frequently noted for the first 6 months of life, but referral is indicated if it persists beyond 6 months.
A 3-year-old girl presents to the ED with acute-onset high fever, drooling, refusal to eat, difficulty breathing, and a muffled voice. She and her family are recently-arrived immigrants who have been living in a refugee camp in Uganda for the last two years. She has no significant past medical history. On exam, you note an ill and anxious-appearing child who is sitting up and leaning forward on the stretcher, with her head up, neck extended, and an audible soft stridor. Which pathogen is most likely causing her illness? A) Haemophilus influenzae type b B) Parainfluenza virus C) Pseudomonas aeruginosa D) Staphylococcus aureus
A) Haemophilus influenzae type b (MC cause of infectious epiglottitis)
An 8-year old male presents with pain in his legs after exercise. Physical examination reveals radial-femoral pulse delay and the blood pressure in the lower extremities is 15 mmHg lower than in the upper extremities. Which of the following lab or radiological findings would be most commonly noted in this patient? A) Inferior border rib notching B) Decreased hemoglobin C) Left atrial enlargement D) Elevated troponin
A) Inferior border rib notching Coarctation of the aorta presents with a disparity in both pulses and blood pressure between the upper and lower extremities. Laboratory or radiologic findings include inferior border rib notching, cardiomegaly, and ventricular hypertrophy on EKG.
A 9-year-old girl presents to urgent care because a live mosquito became lodged in her ear today. She is uncomfortable, but hearing is normal. Exam reveals a moving insect in the ear canal with intact tympanic membrane. Her mother denies any history of ear problems, including prior tympanostomy tube placement. Which of the following interventions is the most appropriate first step in management? A) Irrigation with warmed mineral oil B) Irrigation with warmed normal saline C) Removal under visualization with alligator forceps D) Urgent ENT consultation
A) Irrigation with warmed mineral oil
A 14-year-old boy presents with one week of an itchy rash over his back. On examination, there are multiple oval-shaped papules in linear distributions over his upper back. There is also a prominent oval-shaped plaque on his upper back. This larger plaque has a small area of central clearing and fine scale. What is the most likely diagnosis? A) Pityriasis rosea B) Secondary syphilis C) Tinea corporis D) Tinea versicolor
A) Pityriasis rosea
Influenza Vaccine Timing
Annually after 6 months 2 doses for first time at least 1 month apart
A 2-year-old presents with fever and drooling. The patient appears toxic and is sitting up with chin forward. Thumbprint sign is noted on lateral neck x-ray film. What is the treatment of choice for this patient?
Artificial airway Acute epiglottitis is a medical emergency and immediate treatment with an artificial airway in a controlled environment is indicated. Steroids and racemic epinephrine are ineffective and not indicated. The antibiotic of choice is ceftriaxone, or cefotaxime. Ampicillin is not considered first line due to increased resistance of H. influ to ampicillin
At what age should the first hepatitis B vaccine be administered
Birth Hepatitis B vaccine is first given at birth, then at 1 to 2 months, and again at 6 to 18 months
Male Genitalia Tanner Stage 2
Enlargement of the testes and scrotum with redenning and thinning of the scrotum, but no enlargement of the penis
Red Flags in Fine Motor Development
Lack of transfer at 7 mos Using one hand exclusively at any age Delayed self-care (ADLs) at 4 yrs Delayed printing at school entry
Language at 4 Years
Large vocabulary 2000 words Strangers can understand full vocabulary
4-9 Year EPSDT
Monitor development, speech, learning Nutrition and healthy eating habits Bullying, vision (glasses) Psychosocial/behavior assessment Check weight, heigh, BMI, BP
What is the leading cause of injury-related death in children between the ages of 1 and 15?
Motor vehicle injuries Motor vehicle injuries are the leading cause of death in children.
An afebrile 2 year-old female presents with a three-day history of foul smelling, blood-tinged, mucoid drainage from the left nostril. What is the most likely diagnosis in this patient?
Nasal foreign body Nasal foreign body typically presents in children under 3 years of age. The symptoms include mucopurulent drainage, epistaxis, foul odor and nasal obstruction.
Skills of Gross Motor Development
Posture Change of position Ambulation Strength Balance Coordination
Breast Tanner Stage 1
Prepubertal breast where there is only elevation of the papilla
Trends in Gross Motor Development
Primitive reflexes to volitional movement Cephalo-caudal Proximal to distal Flexor to extensor Postural reflexes fade as balance increases Single movements to sequences Effortful to automatic
6 Month EPSDT
Review development (sits +/- support, transferring objects, peek-a-boo, babbles, goes from sitting to all 4s, floor time) May start solids (veggies to fruits, 1 new food every 3-5 days, high chair use) If on premature formula: assess need to continue Review recommended vaccines Schedule 9 month EPSDT
24 Month EPSDT
Review development (speaks 20 words, learning new words daily, walking, running well, potty training, emotional, says "no" a lot, two word phrases, walks on tiptoes) Dental referral Refer to AZEIP prn (especially for speech concerns) CBC/Lead, MCHAT Review recommended vaccines
A 3 year-old boy is seen in the office with a 5-day history of fever, erythema, edema of the hands and feet, a generalized rash over the body, bilateral conjunctival injections, fissuring and erythema of the lips, and cervical adenopathy. Antistreptolysin A (ASO) titer and throat culture are negative. The most serious systemic complication associated with this disorder is
cardiac The patient most likely has Kawasaki syndrome. The major complication with this disorder is coronary artery aneurysms, which are reported in up to 20% of affected children. The etiology of this disorder is uncertain, although a bacterial toxin with super antigen properties may be involved.
A 3-year-old boy is seen in the office with a 5-day history of fever, erythema, edema of the hands and feet, a generalized rash over the body, bilateral conjunctival injections, fissuring and erythema of the lips, and cervical adenopathy. Antistreptolysin A (ASO) titer and throat culture are negative. The most serious systemic complication associated with this disorder is
cardiac. The patient most likely has Kawasaki syndrome. The major complication with this disorder is coronary artery aneurysms, which are reported in up to 20% of affected children. The etiology of this disorder is uncertain, although a bacterial toxin with super antigen properties may be involved.
The most definitive treatment for primary enuresis is
desmopressin (DDAVP) Intranasal desmopressin is effective in 50% of patients treated and is the treatment of choice.
A 2-year-old presents with sudden onset of cough and stridor. On examination the child is afebrile and appears well with a respiratory rate of 42 per minute. What is the next best step in the evaluation and treatment of this patient?
indirect laryngoscopy Laryngoscopy is indicated not only for diagnosis, but also removal of the foreign body.
A 13 year-old boy with leukemia presents with epistaxis for 2 hours. The bleeding site appears to be from Kiesselbach's area. The most appropriate intervention is
intranasal petrolatum gauze. Petrolatum gauze will provide pressure to the bleeding point while the cause of bleeding is corrected.
A 15-year-old male was seen last week with complaints of a sore throat, headache, and mild cough. A diagnosis of URI was made and supportive treatment was initiated. He returns today with complaints of worsening cough and increasing fatigue. At this time, chest x-ray reveals bilateral hilar infiltrates. A WBC count is normal and a cold hemagglutinin titer is elevated. The most likely diagnosis is
mycoplasma pneumonia. The insidious onset of symptoms, the interstitial infiltrate on chest x-ray, and elevated cold hemagglutinin titer make this diagnosis the most likely.
A 16 day-old male presents in the office with a history of vomiting after feeding for the past 2 days. The vomiting has become progressively worse and the mother describes it as very forceful, sometimes hitting the floor 6 feet away. She says the neonate is always hungry. On physical examination, it is noted that he is 2 ounces below birth weight, and has a small palpable mass (about 1.5 cm) in the epigastrium. The most likely diagnosis is
pyloric stenosis. Pyloric stenosis begins between 2 to 4 weeks of age with vomiting that becomes projectile after each feeding. An olive-size mass can often be felt in the epigastrium.
A mother brings in her 2 year-old child stating that the child has had a 3-day history of a nonproductive cough, thick copious rhinorrhea, conjunctivitis, and a fever to 103 degrees. Physical examination reveals a well-hydrated child, with numerous 1-2 mm white papules on both buccal mucosa, normal heart and breath sounds. This presentation is most consistent with early
rubeola. Rubeola (measles) is characterized by cough, coryza, and conjunctivitis, along with a fever as a prodrome. Koplik spots appear prior to the onset of the typical erythematous, maculopapular rash and are pathognomonic for rubeola.
A 14 year-old male who is overweight presents with complaints of left knee and anteromedial thigh pain for the past month. He states the pain gets better with rest and denies any known trauma. On examination of the gait, a slight limp is noted. X-ray films of the left knee are normal. The most likely diagnosis is
slipped capital femoral epiphysis. Slipped capital femoral epiphysis is most common in overweight adolescent males who present with complaints of pain that is referred to the thigh or medial side of the knee associated with a limp. X-ray films of the knee are normal since the condition involves the hip.
A child has been under treatment for attention-deficit hyperactivity disorder (ADHD). No response has occurred with behavioral adaptations. Which category of medication should this patient be given?
stimulants Stimulants, such as Ritalin, Dexedrine, and Cylert, are effective in 50 to 80% of children with ADHD.
Hepatitis B Vaccine Timing
1. Birth 2. 6 weeks to 2 months 3. 6 months
A newborn weighs 8 pounds at birth. On average, what should the infant weigh at 1 year of age?
24 pounds An infant will triple birth weight within the first year of life. A newborn that weighs 8 pounds at birth will weigh approximately 24 pounds at 1 year of age.
Hearing screen should be done at
3 years and up
An 8-year-old boy presents to his pediatrician with a low-grade fever, new rash, and pruritus. He has not been vaccinated for varicella and mom confirms a recent exposure. On exam, a pink vesicular rash in various stages of development is noted, as well as surrounding linear excoriations. There are no other presenting complaints, and he has no significant past medical history. Which of the following is the most appropriate therapy? A) Acetaminophen B) Acyclovir C) Aspirin D) Valacyclovir
A) Acetaminophen (varicella is self-limited so this is the most appropriate for symptom management)
A 9-month-old baby boy presents to the emergency department with a chief complaint of worsening cough. His mother notes that soon after he was born, he developed respiratory distress and was admitted to the neonatal intensive care unit for a month. He was initially breastfed, but due to frequent vomiting and loose bowel movements, formula feeding was given. Despite the formula feeding he still developed diarrhea and failure to thrive. He is also noted to have loose, large, greasy, and foul-smelling stools. For the past three days his mother notes increased cough and sputum production, increased fatigue and decreased appetite. Physical examination reveals an alert and active baby boy in mild respiratory distress with temperature 37.8°C, pulse rate 120 beats per minute, respiratory rate 65 breaths per minute, pulse oximetry 91% on room air, and weight for age less than the fifth percentile. He has anicteric sclerae, pink palpebral conjunctivae, bilateral otitis media, mild nasal congestion, no cervical lymphadenopathy, lungs with scattered wheezing and rales, adynamic precordium, no heart murmur, soft non-tender abdomen, full equal pulses, and capillary return time of less than two seconds. Which of the following is the best treatment for the boy? A) Advise chest physiotherapy B) Discuss lung transplantation C) Give palivizumab D) Prescribe cromolyn
A) Advise chest physiotherapy (first-line treatment for cystic fibrosis)
A 4-year-old girl presents to the emergency department with two days of nasal congestion. Tonight, she developed chest pain and difficulty breathing. She has increased work of breathing, tachypnea, scattered soft wheezes, and poor air movement at the bilateral lung bases. Which of the following is the most appropriate initial medication? A) Albuterol B) Magnesium sulfate C) Prednisolone D) Terbutaline
A) Albuterol (TOC for asthma exacerbation)
A 9-year-old boy has been diagnosed with stage III non-Hodgkin lymphoma. It is anticipated that he will develop nausea and vomiting during chemotherapy. What class of medication is recommended prior to starting treatment to decrease these side effects? A) Benzodiazepines B) Glucocorticoids C) Neurokinin-1 receptor antagonists D) Three 5-hydroxytryptamine receptor antagonists
A) Benzodiazepines (TOC for pretreatment) *three 5-hydrocytrptamine receptor antagonists are TOC for acute treatment*
Which of the following best describes the effects of fetal coarctation of the aorta in utero? A) Blood flow through the patent ductus arteriosus bypasses the coarctation, resulting in hemodynamic stability B) Decreased blood flow distal to the coarctation causes growth restriction of the lower extremities C) Decreased blood flow distal to the coarctation leads to renal insufficiency and oligohydramnios D) Increased aortic pressure proximal to the coarctation causes left ventricular hypertrophy
A) Blood flow through the patent ductus arteriosus bypasses the coarctation, resulting in hemodynamic stability
A 4-year-old presents with a history of having failed two courses of antibiotic therapy for acute otitis media. Initially she was on high-dose amoxicillin for 10 days followed by amoxicillin-clavulanate (Augmentin). Mother has been compliant with administering the medication. Which of the following is the next most appropriate intervention? A) Typanocenntesis B) ENT referral for tubes C) repeat course of Augmentin D) antibiotic and corticosteroid therapy
A) Tympanocentesis This child has unresponsive acute otitis media having failed two courses of appropriate antibiotic therapy. Tympanocentesis is indicated to identify the causative organism and appropriate antibiotic selection.
Functions Affected by Adult Loss of Social/Emotional Development
Adaptive functioning
Upon stroking of the lateral aspect of the sole from the heel to the ball of the foot, the great toe dorsiflexes and the other toes fan. This is a positive
Babinski's sign. A Babinski test is performed by stroking the lateral aspect of the sole from the heel to the ball of the foot, the great toe dorsiflexes and the other toes fan in a positive test.
Trends in Social/Emotional Development
Bonding: parent > child Attachment: child > parent Increasing social circle: self > primary caregiver > extended family > peers > broader community > country and culture > world
Evaluation of Short Stature
Bone age imaging CBC, CMP, TSH, IGF-1, IGFBP-3, GH, ESR/CRP, EMA IgA and IgG, tTG IgG, DGP IgG, Iron-panel, prealbumin, sweat chloride Karotype for Turner's Syndrome
A 2-month-old boy presents to the clinic for his vaccinations. On exam, a continuous 3/6 murmur is noted in the left infraclavicular region. Which of the following would be the most appropriate diagnostic study to order in this patient? A) Angiography B) Chest X-ray C) Echocardiogram D) Electrocardiogram
C) Echocardiogram
CRAFFT
Car Reflex Alone Forget Friends Trouble
A 24 year-old male presents for routine physical examination. On physical examination, you find that the patient's upper extremity blood pressure is higher than the blood pressure in the lower extremity. Heart exam reveals a late systolic murmur heard best posteriorly. What is the most likely diagnosis in this patient?
Coarctation of the aorta Coarctation of the aorta commonly presents with higher systolic pressures in the upper extremities than the lower extremities and absent or weak femoral pulses.
4 Month EPSDT
Confirm patient has doubled birth weight May start rice/oatmeal cereal Review development (holding head up, rolling over, playful, active, laughs, holds up head with tummy time, turns head in response to voice, drooling) Consult nutrition Review recommended vaccines Schedule 6 month EPSDT
An infant is born and has an initial Apgar score completed 1 minute after delivery. When should the next Apgar score be completed? A) 2 minutes after birthY B) 3 minutes after birth C) 4 minutes after birth D) 5 minutes after birth
D) 5 minutes after birth A-activity P-pulse G-grimace A-appearance R-respirations 7-10: considered normal 4-7: potential for resuscitative measures <3: need for resuscitation
Red Flags in Social/Emotional Development
Emotional dysregulation Abnormal attachment patterns Limited social smiling and shared enjoyment by 6 mos Limited gestures like pointing, response to name, joint attention by 12 mos Limited social imitative play by 18 mos Limited pretend play by 24 mos No friends at school-age
Breast Tanner Stage 5
Fully mature with recession of the secondary mound and a smooth breast contour
Pubic Hair Tanner Stage 3
Hair increases in quantity and is darker and curlier
Pubic Hair Tanner Stage 1
May be fine vellus hair that is no different from that found over the abdominal wall
Who Assesses Fine Motor Development
PCP OT
Trends in Fine Motor Development
Primitive reflexes to volitional movement (grasp) Proximal to distal (whole hand to fingertips) Effortful to automatic Unilateral to bimanual coordination
A foreign body lodged in the trachea that is causing partial obstruction will most likely produce what physical examination finding?
Stridor An inspiratory wheeze is called stridor, which indicates a partial obstruction of the trachea or larynx.
12-15 Month EPSDT
Switch from formula to cow, soy, etc. D/c pacifiers, bottles (Sippy cups for all liquids) No juice until 2 yrs Review development (walking, saying 5-10 words, feeds self, tantrums, growing independence) CBC/Lead, dental referral Review recommended vaccines Schedule 15 month EPSDT
Male Genitalia Tanner Stage 4
Testes and scrotum continue to enlarge with continued lengthening of the penis and enlargement of the glans
A 3 week-old infant is evaluated for persistent projectile vomiting described as breast milk without bile or blood. The abdomen is distended before vomiting and a small, mid-epigastric mass is palpable after vomiting. What is the most appropriate diagnostic study for the evaluation of this patient?
Upper GI contrast radiographs This infant has pyloric stenosis and an upper GI series will reveal a narrowed distal stomach with double tract of barium.
What is a common side effect of the psychostimulants?
Weight loss The side effects of the psychostimulants, such as Ritalin, include appetite suppression, weight loss, and sleep disturbances
A 2 month-old infant presents for a routine health maintenance visit. The mother has been concerned about the infant's hearing since birth. Physical examination reveals no apparent response to a sudden loud sound. Which of the following is the most appropriate diagnostic evaluation? A) pneumonia B) bronchiolitis C) croup D) asthma
auditory-evoked potentials Brainstem auditory-evoked potentials evaluate the sensory pathway and identify the site of any anatomical disruption. The test does not require any active response from the patient and is useful in the evaluation of suspected hearing loss in an infant.
A 13 year-old child presents with a lesion on his right forearm that is occasionally pruritic. On examination, a 2 cm ring of erythema with a scaly border and central clearing is noted. A suspected diagnosis is confirmed by the presence of
hyphae on a KOH prep. Ring-shaped lesions with scaly borders and central clearing are most likely caused by fungal infection. Microscopic examination of scrapings reveals hyphae on KOH prep.
A 7-year-old child with a history of type 1 diabetes mellitus for 3 years presents for routine follow-up. The mother states that the child has been having nightmares and night sweats. Additionally, his average morning glucose readings have risen from an average of 100 mg/dL to 145 mg/dL over the past week. This child is most likely experiencing
the Somogyi effect. This refers to nocturnal hypoglycemia, which stimulates counter-regulatory hormone release resulting in rebound hyperglycemia.
Tetanus, Diphtheria, Pertussis (Tdap) Vaccine Timing
1. 11 years 2. Repeat every 10 years
What is the treatment of choice for Kawaski's disease?
Aspirin Patients with Kawasaki's disease present with fever, bilateral conjunctival injection, pharyngeal erythema, edema of the hands and feet, rash, and LAD. Tx of choice is high-dose aspirin and IV immunoglobulin
EPSDT Measurements 3 yrs and up
Blood pressure, vision
The most accurate way to determine the exact degree of spinal curvature in a child with scoliosis is by
Calculation of the Cobb angle The scoliotic curve is measured by the Cobb method using AP and lateral x-ray films of the entire length of the spine.
A 3 week-old male infant presents with recurrent regurgitation after feeding that has progressed to projectile vomiting in the last few days. The mother states that the child appears hungry all of the time. She denies any diarrhea in the child. What clinical finding is most likely?
Olive-sized mass in the right upper abdomen An olive-sized mass may be palpated in the right upper abdomen in pyloric stenosis and if found, is pathognomonic for pyloric stenosis.
Polio (IPV) Vaccine Timing
1. 6 weeks to 2 months 2. 4 months 3. 6 months 4. 4 years
Which of the following is the most common cause of hypertrophic cardiomyopathy in the newborn? A) Inborn errors of metabolism B) Multiple congenital anomaly syndromes C) Mutations of sarcomeric proteins D) Neuromuscular disorders
C) Mutations of sarcomeric proteins
Small grayish vesicles and punched-out ulcers in the posterior pharynx in a child with pharyngitis is representative of which organism?
Coxsackievirus Coxsackievirus presents with small grayish vesicles and punched-out ulcers in the posterior pharynx.
Who is the most likely adult to sexually abuse a child?
Family member Adults within the immediate or extended family perpetrate the most child sexual abuse, usually this is a trusted member of the family.
Areas of Development
G: Gross Motor F: Fine Motor S: Speech/Language C: Cognitive/Problem Solving S: Social/Emotional
Developmental Milestones at 4 yrs
G: Hops on one foot, down stairs alternating feet F: Draws x/square/diagonals, cuts shape with scissors, buttons S: Sentences 100% intelligible, tells a story, past tense C: Counts to 4, opposites, identifies 4 colors S: Has preferred friend, elaborate fantasy play
A 12 year-old female presents for a routine sports physical. The physical exam reveals asymmetry of the posterior chest wall on forward bending. This is most consistent with
Scoliosis Asymmetry of the posterior chest wall on forward bending is the most striking and consistent abnormality in patients with idiopathic scoliosis.
Male Genitalia Tanner Stage 3
The penis begins to enlarge, first in length and later in diameter Testes and scrotum continue to enlarge
What is the treatment of choice for homocystinuria?
Vitamin B6 Homocystinuria is a disorder of amino acid metabolism and is best treated with high doses of Vitamin B6
What is a milestone usually achieved by a 15-month old infant?
walks alone
An infant presents with bullous impetigo involving the face, extremities, and trunk. What is the best treatment for this child?
oral dicloxacillin. Bullous impetigo that is localized may be treated with topical mupirocin, but extensive involvement is best treated with oral antibiotics that are penicillinase-resistant, such as dicloxacillin.
A six-year-old child presents with circular patches of baldness and hair that fluoresces yellow-green under a Woods lamp. What would be the best treatment?
oral griseofulvin Tinea capitis is best treated with oral griseofulvin until the lesions clear, usually 4-8 weeks.
Pneumococcal Conjugate (PCV13) Vaccine Timing
1. 6 weeks to 2 months 2. 4 months 3. 6 months 4. 12 months
Diphtheria, Tetanus, Pertussis (DTaP/DT) Vaccine Timing
1. 6 weeks to 2 months 2. 4 months 3. 6 months 4. 12 months (6 or > between 3rd and 4th dose) 5. 4 years
What is the treatment of choice for stage one Lyme disease in a patient less than 8 years of age?
Amoxicillin Amoxicillin or cefuroxime are first-line therapy in a patient less than 8 years of age due to the harmful effects of doxycycline on teeth and bones in children.
A 2 year-old female presents with purulent nasal discharge bilaterally with fever and cough for several days. Her mom had taken her out of daycare for a similar occurrence 2 months ago, that was treated with Amoxicillin. Exam further reveals halitosis and periorbital edema. Treatment should be initiated with
Amoxicillin-clavulanate (Augmentin) High dose amoxicillin-clavulanate is the treatment of choice for resistant bacterial sinusitis, especially in children presenting with risk factors (daycare attendance, previous antibiotic treatment 1-3 months prior, age younger than 2 years).
Murmurs heard best over the Apex
Aortic Stenosis HOCM Mitral Regurgitation Mitral Valve Prolapse Still's
Murmurs heard best over the RUSB
Aortic Valve Stenosis Stills
Examples of Adult Functional Loss of Speech/Language Development
Aphasias
Breast Tanner Stage 4
Areola forms a secondary mound above the contour of the breast
Which of the following referrals would be beneficial for a child with cystic fibrosis? A) Cardiac rehabilitation B) Chest physiotherapy C) Gastroenterology for feeding tube insertion D) Nephrology for hemodialysis
B) Chest physiotherapy
A 12 month-old child with tetralogy of Fallot is most likely to have which of the following clinical features? A) chest pain B) cyanosis C) convulsions D) palpitations
B) Cyanosis The main characteristic of tetralogy of Fallot is cyanosis. Hypercyanotic spells or "tet spells" are paroxysmal episodes in which the cyanosis acutely worsens. Crying, feeding, or defecating can bring on these episodes.
A mother brings her 6 year-old boy for evaluation of school behavior problems. She says the teacher told her that the boy does not pay attention in class, that he gets up and runs around the room when the rest of the children are listening to a story, and that he seems to be easily distracted by events outside or in the hall. He refuses to remain in his seat during class, and occasionally sits under his desk or crawls around under a table. The teacher told the mother this behavior is interfering with the child's ability to function in the classroom and to learn. The mother states that she has noticed some of these behaviors at home, including his inability to watch his favorite cartoon program all the way through. What is the most likely diagnosis?
Attention deficit hyperactivity disorder Attention deficit hyperactivity disorder is characterized by inattention, including increased distractibility and difficulty sustaining attention; poor impulse control and decreased self-inhibitory capacity; and motor overactivity and motor restlessness, which are pervasive and interfere with the individual's ability to function under normal circumstances.
A 12-year-old girl presents to the primary care physician for her first physical following her international adoption. She is feeling well, but her mother was recently diagnosed with fulminant hepatitis B. Which of the following would indicate the child is immune to hepatitis B due to vaccination? A) HBsAg negative and anti-HBs negative B) HBsAg negative and anti-HBs positive C) HBsAg positive and HBeAg negative D) HBsAg positive and HBeAg positive
B) HBsAg negative and anti-HBs positive
EPSDT Measurements 24 mos and up
BMI
A boy presents to clinic for a well-baby check. The mother notes that her child is able to move his head from side to side following a moving object, can lift his head from a prone position, smiles when played with, and makes cooing sounds. The physical examination reveals a calm, afebrile, baby boy with pink conjunctivae, clear breath sounds, adynamic precordium, distinct heart sounds, no heart murmurs, soft non-tender abdomen, full equal pulses and no edema. He is unable to maintain a seated position. Which of the following is the most likely age of the infant? A) 1 month B) 12 months C) 4 months D) 7 months
C) 4 months
Which of the following is the most appropriate intervention in suspected child abuse? A) Arrange for the arrest of the parents B) Confront the suspected assailant in front of the child C) Assure the safety of the child, with hospitalization if necessary D) Contact social service department after discharge of the child
C) Assure the safety of the child, with hospitalization if necessary. The primary goal should be the safety and well-being of the child. Hospitalization may be the only way the clinician has to remove the child from the care of a possible abusive home if no other recourse is available due to a lack of social services and investigators.
A 17-year-old girl presents with the complaint of a skin rash. She began treatment two days ago for a urinary tract infection and was given sulfamethoxazole-trimethoprim. On physical exam, bullae are seen covering 40% of her body and are present in her mouth as well. What is the best step in managing the patient? A) Continue sulfamethoxazole-trimethoprim B) Lower the room temperature to 21°C C) Refer to a burn center D) Start antibiotic prophylaxis against Pseudomonas aeruginosa
C) Refer to a burn center (TOC for TEN) -offending drug discontinued and supportive care similar to major burns
A 12-year-old boy presents with an itchy rash on his hands and wrists. Several of his family members have recently been treated for a similar rash. Physical examination reveals papules and excoriations on his fingers and hands, particularly in the web spaces of his digits. Which of the following is the best pharmacologic treatment for this patient? A) Oral mebendazole B) Topical lindane cream C) Topical permethrin cream D) Topical triamcinolone cream
C) Topical permethrin cream (TOC for scabies)
A 2 year-old child is brought to the office because of a cough and a fever of 102 degrees F for 2 days. The physician assistant notes the presence of hoarseness, a barking cough, and stridor. The ears and nose exam are unremarkable. Auscultation of the chest reveals decreased breath sounds without crackles or expiratory wheezes. What would be the initial diagnostic impression?
Croup Hoarseness, inspiratory stridor, and a barking cough are classic signs of croup, all of which are noted in this patient.
A 13 year-old female presents to the office with right knee and thigh pain and the inability to bear weight since waking yesterday morning. The mother states the child had a fever of 100.9 degrees F this morning and continues to be non- weight bearing. Examination reveals a warm, erythematous, swollen knee. What tests would be most beneficial in the diagnosis and treatment of this patient?
Culture of joint aspirate A culture of the joint fluid will confirm the diagnosis and offer information regarding infectious agent.
Which of the following is a common symptom associated with laryngotracheobronchitis (viral croup)? A) Drooling B) High fever C) "Hot potato" voice D) Barking cough
D) Barking cough Viral croup is characterized by a history of upper respiratory tract symptoms followed by onset of a barking cough and stridor.
Which of the following is an indication for a pediatric patient to receive the 23-valent polysaccharide vaccine (Pneumovax)? A) Children at any age with a history of asthma B) All children at 2, 4, 6, and 12-18 months of age C) All children at 12-23 months of age in a 2 dose series D) Children age 24-59 months at high risk for invasive pneumococcal disease
D) Children age 24-59 months at high risk for invasive pneumococcal disease Pneumovax is licensed for use in children over the age of 23 months and is indicated for all pediatric patients at increased risk for pneumococcal disease.
A 6-year-old male presents with hemarthrosis of the left knee. Coag studies: PT 12.5s (normal12-14 sec), INR 1.0, aPTT 58s (normal 18-28 sec), platelet 430,000/microliter (normal 150,000-450,000/microliter), and bleeding time 4m (normal 2-12m). Which is the best treatment option for this patient? A) Desmopressin acetate B) Corticosteroids C) Vitamin K D) Cryoprecipitate
D) Cryoprecipitate Hemophilia A presents with a prolonged aPTT and normal platelet count and function. Hemophilia A is treated with factor VIII concentrate or cryoprecipitate.
A two-day-old, full-term female with a history of breech presentation and delivery by cesarean section has an unremarkable newborn examination, including a normal bilateral hip exam. There is no family history of developmental dysplasia of the hip reported. Given this patient's presenting risk factors, which of the following is the most appropriate plan for follow-up of her hip exam? A) Hip examination with ultrasound for any abnormalities at every health visit B) Hip X-ray at four to six months of age C) No special follow-up unless patient has delayed achievement of developmental milestones D) Ultrasound of the hips at four to six weeks of age
D) Ultrasound of the hips at four to six weeks of age
A previously healthy 10-year-old boy presents to the clinic with his mother because she is worried he has an infection. He has had a fever and malaise for five days for which he was given antipyretics. On physical examination his vital signs are within normal limits for age except for a low-grade fever. Which of the following is most consistent with mumps virus infection? A) Gradual bilateral swelling of the parotid glands B) Sudden onset of firm, erythematous swelling of the parotid gland C) Unilateral painless parotid swelling D) Unilateral parotid swelling and tenderness that becomes bilateral
D) Unilateral parotid swelling and tenderness that becomes bilateral
An examination of the hips of a newborn, a "clunk" is noted with abduction of the right hip to almost 90 degrees while lifting the greater trochanter. What is the most likely diagnosis?
Developmental dysplasia of the hip The Ortolani test is positive in the newborn patient with developmental dysplasia of the hip. The test is performed by lifting the greater trochanter while abducting the hip to 90 degrees.
In the neonate, unequal thigh folds may indicate what?
Developmental hip dysplasia A dislocated hip displaces proximally in developmental hip dysplasia, causing a shortening of the leg that may present as unequal thigh folds.
Examples of Speech/Language Developmental Disorder
Developmental language disorders Language-based learning disability (dyslexia) Dysarthria/stuttering
A 12-year-old boy presents with fever and severe sore throat. What physical exam findings helps differentiate a peritonsillar abscess from epiglottitis?
Deviation of the uvula While epiglottitis and peritonsillar abscess share several similar presenting signs and symptoms, the key differentiating finding is a uvular deviation which is only found with a peritonsillar abscess. Peritonsillar abscesses occur primarily in young adults. This condition most often follows Group A streptococcal pharyngitis and exudative tonsillitis. The abscess is polymicrobial and generally presents with fever, malaise, sore throat, dysphagia, and otalgia. On physical exam, the patient may have trismus and a muffled voice (classically called a "hot potato voice").
13-18 Year EPSDT
Fasting labs: CBC, CMP, TSH, T4, Lipid panel STD screening: urine GC/chlamydia, RPR, HIV (16-18), Hepatitis panel (15-17 yrs) Depression screening Review recommended vaccines
Developmental Milestones at 9 mos
G: Gets from all 4s to sitting, sits well with hands free, pulls to stand, creeps on hands and knees F: inferior pincer grasp, pokes at objects S: Specific babble (mama/dada), gestures bye and up, gesture games C: Object permanence, uncovers toy, "Peek-a-boo" S: Separation anxiety
Developmental Milestones at 2 yrs
G: Jumps on 2 feet, up and down stairs while "marking time" F: Handedness established, uses fork, tower of 6 locks, imitates vertical stroke S: Follows 2-step command, 50+ words with 50% intelligible, 2-word phrases, "I"/"me"/"you"/plurals C: New problem-solving strategies without rehearsal, searches for hidden object after multiple displacements S: Testing limits, tantrums, negativism, possessive
Developmental Milestones at 3 yrs
G: Peals trike, up stairs alternating feet F: Undresses, toilet trained, draws circle/cross, turns pages of books S: 3-step commands, 200 words 75% intelligible, 3-4 word phrases, W questions, states full name/age/gender C: Simple time concepts, identifies shapes, compares 2 items, counts to 3 S: Separates easily, sharing, empathy, cooperative play, role play
Developmental Milestones at 6 mos
G: Postural reflexes, sits tripod, rolls both ways F: Raking grasp, transfers hand to hand S: Babble (nonspecific) C: Stranger anxiety, looks for dropped or partially hidden object S: Expresses emotions, memory lasts 24 hrs
Developmental Milestones of the Newborn
G: Primitive reflexes (step, place, Moro, Babinski, ATNR), Flexor posture F: Primitive reflexes (grasp) S: Primitive reflexes (root, suck), alerts to sound, startles to loud sounds, variable cries C: Visual focal length 10 degrees, fix and follow slow horizontal arc, prefers contrast/colors/faces, prefers high pitched voice S: Bonding, self-regulation/soothing
Developmental Milestones at 4 mos
G: Sits with support, head up to 90 prone with arms out, rolls front to back F: Palmar grasp, reaches and obtains items, brings objects midline S: Laugh, razz, "ga", squeal C: Anticipates routines, purposeful sensory exploration of objects S: Turn-taking conversations, explores parent's face
Developmental Milestones at 18 mos
G: Stoops and recovers, runs F: Carries toys while walking, removes clothing, tower of 4 blocks, scribbles with fisted pencil grasp S: Points to objects, can pick out 3 body parts, 10-25 words, embedded jargoning, labels familiar objects C: Irritates housework, symbolic play with doll or bear S: Increased independence, parallel play
Developmental Milestones at 12 mos
G: Walks a few steps, wide-based gait F: Fine pincer (fingertips), voluntary release, throws objects, finger-feeds self cheerios S: 1 word with meaning, inhibits with "no!", responds to own name, 1-step command with gesture C: Cause and effect, trial and error, imitates gestures and sounds, uses objects functionally S: Explore from secure base, points at wanted items, narrative memory begins
Developmental Milestones at 15 mos
G: Walks well F: Uses spoon, open top cup, tower of 2 blocks S: Points to 1 body part, 1-step command without gesture, 5 words, Jargoning C: looks for moved hidden object if saw it being moved, experiments with toys to make them work S: Shared attention, points at objects to show parents, brings toys to parent
You are called to the nursery to see a male infant, born by uncomplicated vaginal delivery. He weighs 2,600 grams and has one deep crease on the anterior third of each foot. Respirations are 88 breaths/minute with expiratory grunting and intercostals retractions. He is cyanotic on room air and becomes pink when placed on 60% oxygen. Chest x-ray shows atelectasis with air bronchograms. What is the most likely diagnosis?
Hyaline membrane disease Hyaline membrane disease is the most common cause of respiratory distress in the premature infant. The infant typically presents with tachypnea, cyanosis and expiratory grunting. A chest x-ray reveals hypoexpansion and air bronchograms.
A 16 year-old athlete with no past medical history collapses after running 50 yards down the field. He is unresponsive, pulseless and cyanotic. What is the most likely cause of this student's collapse?
Hypertrophic cardiomyopathy This presentation is consistent with hypertrophic cardiomyopathy which may initially be difficult to diagnose.Infants but not older children frequently present with signs of CHF. Older children may be asymptomatic, with sudden death as the initial presentation
A 9 year-old boy who has had cold-like symptoms for the past few days is brought to the clinic by his mother who states that her son had gross hematuria this morning. Prior to the cold-like symptoms the boy has been in excellent health. He is up-to-date on all of his immunizations. The patient does not have any edema, hypertension or purpura. Urinalysis reveals the urine to be cola-colored with a 2+ positive protein and 2+ hemoglobin. Microscopic analysis reveals 50-100 RBCs/HPF, no WBCs, bacteria, casts or crystals. What is the most likely diagnosis?
IgA nephropathy IgA nephropathy presents after an upper respiratory illness with deposition of IgA within the mesangium of the glomerulus.
A 4 year-old boy presents to the ED after sustaining a crush injury to his distal third phalanx. Physical exam reveals an associated nail bed injury. What is the appropriate management?
Immobilize, antibiotics, orthopedics referral Distal phalanx fracture should be immobilized and if there is an associated nailbed injury the fracture is considered "open" and the patient should be given antibiotics and follow-up with ortho in one week.
Trends in Cognitive/Problem Solving Development
Implicit memory before explicit Specific > general > specific Increased number of dimensions attended to simultaneously "Trial and error" to mental rehearsal Concrete to abstract Increased use of executive functions Increased length of attention span "Metas"
Jones Major Criteria
J- joints O (heart)- cardiac N- nodes E- erythema marginatum S- syndenham chorea
Red Flags in Cognitive/Problem Solving Development
Lack of developmentally appropriate response to visual stimuli Immature play Stereotypic play; lack of pretend School failure *always check vision and hearing if any concerns*
An 8-year-old male presents to a geneticist for further workup of a series of developmental delays, learning disabilities, and dysmorphic features. His past medical history is significant for delays in motor development, including sitting without support at 10 months and walking at 20 months. Academically, he is currently struggling with math in school. On exam, he displays a long, thin, face, a large jaw, and protruding ears seen here. Given this patient's likely diagnosis, what physical finding is he likely to demonstrate following puberty?
Macroorchidism Given the patient's characteristic physical features of a long face, protruding ears, male sex, and learning disability, he likely has Fragile X syndrome. Following puberty, these individuals characteristically have macroorchidism. Fragile X is caused by CGG trinucleotide expansion within the FMR1 gene located on the X chromosome. Affected individuals have characteristic physical features such as macrocephaly, a large jaw, frontal bossing, and macroorchidism following puberty. Additionally, affected individuals may also exhibit strabismus, connective tissue dysplasia including mitral valve prolapse and hyperextensible joints. They may also exhibit specific behaviors such as attention deficit, hand flapping, hand biting, and averting one's gaze. Most affected males have mild to severe mental retardation.
Examples of Adult Functional Loss of Social/Emotional Development
Mental health disorders Post concussion personality changes
Examples of Cognitive/Problem Solving Developmental Disorder
Mental retardation Learning disabillities
Physical exam findings in a 4-year-old child that include blue sclerae and recurrent fractures indicate
Osteogenesis imperfecta Mild osteogenesis imperfecta presents with blue sclerae, history of recurrent fractures and presenile deafness.
Red Flags in Speech/Language Development
Problems with feeding and/or swallowing Parents suspect hearing loss, babbling stops at >6 mos, lack of response to sound No single words by 15 mos No combos by 24 mos Stutter past 3.5 yrs Idiosyncratic speech, disordered sequence of development Poor intelligibility for age
Pubic Hair Tanner Stage 5
Pubic hair has increased further in volume, spread onto the medial thighs, and taken on characteristic male or female configuration
MCHAT Tool
Questionnaire to screen for Autism Critical questions: 2 (deaf), 5 (fingers in front of eyes), 12 (noises) 2/3 as a yes = a failure Failure = refer to developmental peds or AZEIP Other questions inappropriate, consider other developmental disorders
A 3 year-old male presents with a history of high fever, red watery eyes, rhinitis and cough for the past several days. The parents brought him in today due to development of a rash. They deny a history of childhood immunizations. On examination, the child has a barky cough, fever and clear rhinorrhea and sits listlessly on the exam table. The rash is red and maculopapular located on the face and trunk. Koplik spots are noted on the buccal mucosa. What is the most likely diagnosis?
Rubeola Rubeola or measles is characterized by a prodrome of high fever, conjunctivitis, coryza (rhinitis) and a harsh cough. A maculopapular rash quickly spreads from the face to the trunk while the respiratory symptoms are at the worst.
Skills of Social/Emotional Development
Self-awareness Empathy Relationships Emotional regulation
At what age should a child's eyes be consistently well aligned?
Six months A child's eyes should be consistently well-aligned by five to six months of age.
An obese 12 year-old male presents with a 1 month history of right thigh pain worsened with weight bearing. Examination is normal, however, radiographs reveal a posterior and medial displacement of the femoral head. What is the most likely diagnosis?
Slipped capital femoral epiphysis Slipped capital femoral epiphysis is the posterior and usually medially displacement of the femoral head in overweight adolescents during puberty. The growth plate is susceptible to slippage due to the rapidly thickened cartilage physis secondary to growth hormone, the lack of sex hormone secretion for physis stability, mechanic stress secondary to obesity, and the anatomy and hip mechanics.
A 17 year-old female is seen who has a history of eating large amounts of food at night 3 to 4 times weekly. These episodes are always followed by extreme guilt and either induced vomiting or hours of strenuous exercise. She excels at work and school and maintains many active relationships. Her physical examination reveals a normal BMI and an otherwise normal exam. What is the best initial intervention for this patient?
Start an antidepressant Uncomplicated bulimia generally does not require hospitalization as the physical manifestations encountered in bulimia are generally mild if present at all. Some patients with bulimia have concomitant substance abuse issues but her successful relationships and work activities lessen the probability. Antidepressants, not anxiolytics, have been found helpful in lessening the binge/purge cycles and improving overall well-being even outside of comorbid mood disorders.
A 14 month-old male who attends day care presents with a two-day history of frequent watery stools. His mother states that he had a fever and vomiting the day before but these have resolved. His mother denies pain in the child. The child is mildly dehydrated but otherwise appears well. Stool samples are free of blood and white blood cells. The lab reports no ova or parasites noted in the stool samples. What is the most likely diagnosis?
Viral gastroenteritis Rotavirus is the most common cause of gastroenteritis in children and this is frequently passed in the daycare setting.
An 8 year-old child is brought in by his mother with a two day history of spreading, non-pruritic red rash. The rash was preceded by moderate fever, sore throat and rhinorrhea. Examination reveals a moderately ill appearing child with a fine, macular-papular rash on an erythematous base spread diffusely over the trunk with some accentuation in the skin folds. The face is flush with perioral pallor. There is palpable anterior cervical lymphadenopathy. What is the most appropriate diagnostic study to establish the diagnosis?
Throat culture Scarlatina rash is due to infection with group A strep. A throat culture would be the most appropriate diagnostic study to establish the diagnosis in this patient.
What group is most likely to present with Duchenne's muscular dystrophy?
Toddler-aged males Duchenne's muscular dystrophy, a genetic defect on the short arm of the X chromosome, affects toddler-aged males.
A newborn male is evaluated 30 minutes after birth. He was born at 39 weeks gestation to a 27-year-old primigravid via cesarean section for cervical incompetence. The pregnancy was complicated by gestational diabetes, and the amniotic fluid was clear. Upon delivery, the patient had strong respiratory effort and a strong cry. His Apgar scores at 1 and 5 minutes were 7 and 8, respectively. The patient now is exhibiting increased work of breathing and is becoming progressively more tachypneic. His birth weight is 3,568 g (7 lb 14 oz). His temperature is 99.0°F (37.2°C), blood pressure is 60/44 mmHg, the pulse is 146/min, and respirations are 72/min. On physical exam, the patient is grunting with nasal flaring and subcostal retractions. Breath sounds are decreased at the bases bilaterally. The patient has central cyanosis. His chest radiograph can be seen here. What is the most likely etiology of this patient's presentation?
Transient tachypnea of the newborn This newborn is presenting with increased work of breathing and progressive tachypnea in the first hour of life, which is consistent with a diagnosis of transient tachypnea of the newborn. Transient tachypnea of the newborn (TTN) presents with respiratory distress and marked tachypnea within two hours of delivery. TTN is caused by delayed resorption and clearance of alveolar fluid, which results in mild pulmonary edema. Chest radiography typically demonstrates bilateral perihilar streaking and hyperinflation of the lungs. TTN is benign and usually self-resolves by day two of life, and management is supportive.
3 Week Weight Check
Weight Check Review feeding concerns Confirm current weight > birth weight Review fever precautions (>100.5 go to ED) Begin tummy time Review infant stooling every 2-3 days Review development (startle reflex, eyes follow)
EPSDT Measurements Newborn to 2 years
Weight, height, and head circumference
Speech Referral
When: if not meeting speech milestones Check hearing (refer to Audiology or recheck next visit) If <3 yrs: refer to AZEIP online If >3 yrs: refer to school district, consult BH If severe: refer through AHCCS to speech therapy
A 4-year-old presents with severe pain in the left ear. The tympanic membrane is red, dull, bulging, and immobile. The child has no known drug allergies. The most appropriate first-line drug would be
amoxicillin (Amoxil). Amoxicillin will cover the most common pathogens and is considered first-line therapy.
A 2-year-old child presents to the emergency department with increasing respiratory distress. The mother states that the child had a "cold" 2 weeks ago. Last week the cough progressed and is described as barky in nature, associated with stridor. The child appeared to be getting better, but last night,developed a fever and increased respiratory distress. Physical examination reveals a temperature of 102°F. The child is in moderate respiratory distress. A portable lateral neck x-ray film reveals severe subglottic and tracheal narrowing. What is the most likely diagnosis?
bacterial tracheitis Bacterial tracheitis usually presents following a viral upper respiratory infection, especially laryngotracheobronchitis (croup). It should be suspected when a patient develops high fever and respiratory distress after a few days of apparent improvement or if the patient fails to respond to the usual treatment for croup. The findings of subglottic and tracheal narrowing on the lateral neck x-ray film highly support this diagnosis.
A 12 year-old boy presents to the office with pain in his legs with activity gradually becoming worse over the past month. He is unable to ride a bicycle with his friends due to the pain in his legs. Examination of the heart reveals an ejection click and accentuation of the second heart sound. Femoral pulses are weak and delayed compared to the brachial pulses. Blood pressure obtained in both arms is elevated. Chest x-ray reveals rib notching. What is the most likely diagnosis?
coarctation of the aorta Coarctation is a discrete or long segment of narrowing adjacent to the left subclavian artery. As a result of the coarctation, systemic collaterals develop. X-ray findings occur from the dilated and pulsatile intercostal arteries and the "3" is due to the coarctation site with proximal and distal dilations.
A young child is brought to the clinic because the mother noticed a rash while bathing the child. There is a very red slightly raised eruption on the child's face across both cheeks. The child has been in good health and does not appear ill today. The most likely diagnosis is
erythema infectiosum. Erythema infectiosum is generally asymptomatic, presenting with red papules on the face that coalesce to give a "slapped cheek" appearance.
There is considerable debate about the use of tympanostomy tubes in the management of recurrent otitis media in children. Tympanostomy tube placement has been proven to
improve hearing Hearing is improved with tympanostomy tubes by eliminating middle ear effusion when the tubes are functioning properly.
A 4-year-old patient presents with increasing redness and swelling involving her right eye for the past 2 days. The mother states that the child has become increasingly irritable, less active, and appears to have an increased temperature. The child had a recent "cold" and nasal congestion prior to onset of these symptoms. Examination reveals an ill-appearing 4-year-old child lying quietly on the exam table. Temperature is 102° F. Visual activity is 20/40 in the right eye and 20/30 in the left eye. The right eye reveals mild proptosis and severe erythema, increased warmth, and swelling involving the eye and surrounding tissues. What is the most appropriate diagnostic evaluation?
orbital and sinus CT scan This is the typical presentation of orbital cellulitis. A CT scan of the orbit and sinuses is indicated to check for the presence of a subperiosteal abscess and underlying sinusitis, which is often the cause of orbital cellulitis.
A 10-year-old male presents with pain in his left leg that is worse at night. Aspirin relieves the pain and the patient denies injury. On examination, there is point tenderness over the tibia, and the patient has a slight limp that favors the left leg. Radiographs show a 1 cm radiolucent nidus surrounded by osteosclerosis. The most likely diagnosis is
osteoid osteoma. Osteoid osteoma is a benign tumor in children age 5 to 20, presents with increasing pain, worse at night and relieved by aspirin
A 3 year-old presents with a 24-hour history of diarrhea. The patient is afebrile and the stool is noted to be loose and watery. No blood is noted in the stool. Fecal WBC is negative. What is the most likely diagnosis?
viral gastroenteritis Patients with viral gastroenteritis are afebrile and noted to have loose, watery, non-bloody diarrhea. Fecal WBC is negative
The initial sign or symptom of iron poisoning in a 3 year-old child is usually
vomiting and bloody diarrhea. Iron causes localized necrosis and hemorrhage at the point of contact in the GI system resulting in abdominal pain, vomiting, bloody diarrhea, and hematemesis.
An x-ray reveals a break in the cortex of one side of the ulna shaft without a separation or break of the opposite cortex describes what type of fracture?
Greenstick A greenstick fracture is a break in the cortex of one side of bone shaft without a break in the opposite cortex.
Murmurs heard best over the LLSB
HOCM (increased with standing_ Ventricular Septal Defect AV canal defect Tricuspid Regurgitation
Rotavirus Vaccine Timing
1. 6 weeks to 2 months (do not start after 12 weeks) 2. 4 months 3. 6 months (do not give 3rd after 8 months)
Annual blood pressure determinations should be obtained beginning at the age of
3 years Periodic measurements of blood pressure should be part of routine preventive health assessments beginning at the age of 3 years.
A 3-year-old boy presents to your clinic due to his mother's concerns about the patient having intermittent fevers, fatigue, bruising without injury and a pale appearance. Physical exam reveals a palpable liver and spleen, and pale conjunctiva. Initial laboratory testing reveals elevated white blood cell count and lymphoblasts on the peripheral smear. Which of the following is used to confirm the diagnosis? A) Bone marrow aspiration and biopsy B) Computed tomography of the abdomen C) Liver biopsy D) Ultrasound of the testicles
A) Bone marrow aspiration and biopsy (DOC for leukemia)
While examining a newborn, the PA notes symmetrical abduction of the upper extremities and extension of the fingers when the newborn's head is allowed to drop a few centimeters while supporting the body. This describes which of the following reflexes? A) Moro's B) Galant's C) Placing D) Parachute
A) Moro's Moro's reflex is noted by symmetrical abduction of the upper extremities and extension of the fingers after dropping the head a few centimeters while supporting the body. The parachute reflex is noted by extension of all extremities with forward flexing of the infant as if to fall. Galant's reflex is noted by lateral curvature of the trunk with stroking one side of the back. Placing response is noted by flexion of the knee and hip and placing the foot on the table when allowing the newborn's feet to lightly touch the surface of the table.
A 5-year-old girl presents with two weeks of nasal congestion and low-grade fever. Yesterday, the area surrounding her left eye began to look puffy, and she complained of blurry vision. On examination, proptosis is present. She also complains of pain on examination of extraocular muscle function, but full examination is limited by the degree of periorbital swelling. What is the most likely diagnosis? A) Graves' disease B) Orbital cellulitis C) Orbital neuroblastoma D) Preseptal cellulitis
B) Orbital cellulitis
An 18-month-old girl without significant past medical history presents to the ED with sudden-onset of cough and noisy breathing. Parents report the episode started during dinner. They patted her on the back, and she seemed to improve, but while playing after dinner, the cough and noisy breathing started again and continued to recur in between spells of quiet breathing. She has no recent history of fever, rash, vomiting, upper respiratory infection, and no sick exposures. She is in no acute distress and no apparent pain on presentation but does have an occasional cough. Her physical exam is notable for right-sided wheezing and diminished breath sounds. Which of the following is the most likely diagnosis? A) Acute asthma exacerbation B) Adverse food reaction C) Croup D) Foreign body aspiration
D) Foreign body aspiration
A 3-year-old boy is undergoing a diagnostic work-up for a hereditary cause of a moderate anemia. Lab studies reveal a microcytic anemia and an increased reticulocyte count. Labs also reveal hyperbilirubinemia and an increased mean corpuscular hemoglobin concentration. An osmotic fragility test is abnormal. Which of the following may be eventually considered as a treatment option for this disorder? A) Allogeneic hematopoietic stem cell transplantation B) Avoidance of oxidative drugs C) Oral hydroxyurea D) Splenectomy
D) Splenectomy (treatment option for hereditary spherocytosis)
Examples of Adult Functional Loss of Cognitive/Problem Solving Development
Dementia Specific stroke syndromes
A 7-day-old girl is brought to the general pediatrics clinic by her mother. She was the product of a full-term uncomplicated pregnancy and was delivered via cesarean for breech presentation. The mother received regular prenatal care throughout the pregnancy. This morning, after changing the child's diaper, the mother noticed that the newborn had blood at the vaginal introitus. The mother has no other complaints, and the infant is eating and voiding appropriately. Vital signs are stable. Physical exam reveals moderate mammary enlargement and confirms the vaginal spotting. The remainder of the exam is unremarkable. What is the next step in management?
No tests are needed Vaginal spotting or bleeding is normal in female infants less than 3 months of age and will stop as soon as maternal estrogens are cleared from the baby's blood. The parents of this child only require reassurance. Normal findings in the newborn include non-purulent vaginal discharge, vaginal spotting, and mammary enlargement. All of these symptoms are not a cause for concern and generally regress within a few days. They result from the mother's hormones being transferred to the fetus in utero as well as via breast milk. No workup is required.
A 14-year old male active in sports, has been complaining of intermittent anterior right knee pain for several months. He denies any specific injuries. On examination, there is no erythema, swelling, deformities, joint laxity, or crepitus. Palpation reveals tenderness over the tibial tubercle and bursa of the right knee. This finding is characteristic of which disorder?
Osgood-Schlatter disease Osgood-Schlatter disease causes pain at the tibial tubercle and it is caused by fragmentation of the tip of the proximal tibial physis.
A 12 year-old male presents with pain in his left leg that is worse at night. Aspirin relieves the pain and the patient denies injury. On examination, there is point tenderness over the tibia, and the patient has a slight limp that favors the left leg. Radiographs show a 1 cm radiolucent nidus surrounded by osteosclerosis. What is the most likely diagnosis?
Osteoid osteoma Osteoid osteoma is a benign tumor in children age 5 to 20, presents with increasing pain, worse at night and relieved by aspirin.
Meningococcal Conjugate (MCV) Vaccine Timing
1. 11 years 2. 16 years
Hepatitis A Vaccine Timing
1. 12 month 2. 18 month *at least 6 months between 1st and 2nd dose*
Varicella Vaccine Timing
1. 12 month 2. 4 years
Measles, Mumps, Rubella (MMR) Vaccine Timing
1. 12 months 2. 4 years
Haemophilus influenzae type B (HiB) Vaccine Timing
1. 6 weeks to 2 months 2. 4 months 3. 6 months 4. 12 months
Language at 24 Months
20-50 words 2-word phrases Strangers can understand 50% of language
A 5-year-old boy presents to the office for a follow-up of complaints of pruritus in the perianal area that is worse at night. A cellophane tape test is positive for Enterobius vermicularis. What is the treatment of choice?
Mebendazole (Vermox) Treatment of choice for enterobiasis (pinworms) is mebendazole.
A G1P1 29-year-old mother presents to her 5-week-old son's pediatrician complaining that her son is an "angry" baby and that it is driving her crazy. She is worried that he is never going to become calmer. Over the last couple of weeks, he has been crying for several hours a day on "most days" of the week. She is breastfeeding him every 2-3 hours and changing his diaper after every feed (his urine and stool output is within normal limits). She estimates that he sleeps for 16 hours a day, and when he is awake, she usually carries him in a sling to promote attachment and to provide comfort. On exam, his height, weight, and head circumference are all around the 50th percentile, consistent with his trend since birth. He is alert and active, visually tracks your finger, and has normal muscle tone and reflexes. By what age should this condition improve?
4 months This 5-week-old healthy male infant is crying for at least 3 hours a day, at least 3 days a week, without evidence of a condition that would provoke crying (e.g. - hunger, wet/soiled diaper, fatigue, emotional neglect, organic illness). Thus, he likely has infantile colic, which should resolve by around 4 months of age. Infantile colic usually begins between 2 and 6 weeks of age, with an estimated prevalence of 10-20% of infants. Although the cause is unknown, some theories include: immature gastrointestinal systems or central nervous systems; allergies; sensitive disposition; and maternal misinterpretation of normal crying. Risk factors include smoking, higher socioeconomic status, older parental age, and parental depression.
A 10-day-old neonate is rushed to the emergency department after she developed irritability, diaphoresis, and severe tachypnea. The patient is afebrile and otherwise healthy. She was carried to full term and was delivered vaginally without complications. Temperature is 37°C, pulse rate 185 beats per minute, and respirations 52 breaths per minute. On physical examination, she has a grade 2/6 systolic ejection murmur, loudest at the left upper sternal border with radiation to the left interscapular area. Palpation of the abdomen shows hepatomegaly. Femoral pulses are decreased and her lower extremities appear mottled and mildly cyanotic. Which of the following is the most likely diagnosis? A) Coarctation of the aorta B) Patent ductus arteriosus C) Tetralogy of Fallot D) Transposition of the great arteries
A) Coarctation of the aorta
A 4 year-old child presents with a rapid onset of high fever and extremely sore throat. Which of the following findings are suggestive of the diagnosis of epiglottitis? A) Croupy cough and drooling B) Thick gray, adherent exudate C) Beefy red uvula, palatal petechiae, white exudate D) Inflammation and medial protrusion of one tonsil
A) Croupy cough and drooling A croupy cough with drooling in a patient who appears very ill is consistent with epiglottitis. Examining the throat is contraindicated, unless the airway can be maintained.
Which of the following is the most common risk factor for developing pulmonary nontuberculous mycobacterial disease in children? A) Cystic fibrosis B) DiGeorge syndrome C) Hematopoietic stem cell transplantation D) Hyperimmunoglobulin M syndrome
A) Cystic fibrosis
A full term boy is delivered via Cesarean section to a 34-year-old G1P0 mother. He has good Apgar scores and weighs 2.7 kilograms. Initial physical examination reveals normal findings. An hour after initially breastfeeding, the boy begins to vomit green-tinged milk. He has recurrent bilious vomiting one hour later. On repeat physical examination, he is found to have tachypnea, clear breath sounds and a non-distended abdomen. A nasogastric tube is passed and 30 mL of bilious fluid is aspirated. Which of the following is the most likely diagnosis? A) Duodenal atresia B) Intussusception C) Malrotation D) Pyloric stenosis
A) Duodenal atresia Hallmark sign: bilious vomiting without abdominal distension usually within the first day of life
A 5-year-old girl presents to clinic with a low grade fever and rash. Initially, she had mild nasal congestion, low energy, and decreased oral intake, but then developed a rash that involved both of her cheeks. Now has a fine, lacy rash over her arms and legs. What is the most likely diagnosis? A) Erythema infectiosum B) Hand, foot, and mouth disease C) Roseola D) Scarlet fever
A) Erythema infectiosum
Which of the following therapies is recommended for a 13 month-old child with sickle cell disease? A) Folic acid and penicillin V B) Ferrous sulfate and penicillin V C) Folic acid and ferrous sulfate D) Folic acid, ferrous sulfate and penicillin V
A) Folic acid and penicillin V Patients with sickle cell disease should receive prophylactic penicillin V starting at 2 months of age and folic acid starting at 1 year of age. Ferrous sulfate is not globally recommended for patients with sickle cell disease.
Which of the following results from the deposition of unconjugated bilirubin in the brain? A) Kernicterus B) Rett syndrome C) Neurocysticercosis D) Sturge-Weber syndrome
A) Kernicterus Kernicturus results from the deposition of unconjugated bilirubin in the basal ganglia and brainstem. Rett's syndrome is a neurodegenerative disorder of unknown cause. Neurocysticercosis is caused by infection with a tapeworm. Struge-Weber syndrome is caused by abnormal development of meningeal vasculature
Which of the following is the most appropriate study for diagnosing Hirschsprung disease? A) Rectal biopsy B) Stool leukocyte test C) CT of the abdomen and pelvis D) Fecal occult blood test
A) Rectal biopsy A rectal biopsy showing the absence of ganglion cells in both the submucosal and muscular layers of the involved bowel is the most appropriate diagnostic study for Hirschsprung disease.
A four-year-old boy and his mother present to general pediatrics clinic for follow-up. She is in distress because she has tried everything to get him toilet trained. Her prior child was toilet trained at the age of 2. Her 4-year-old is developmentally appropriate. He can sit upright and can communicate that he needs to use the bathroom. The mother has tried to place him on the toilet at regular intervals and has given him rewards for using the toilet successfully. However, he is not motivated to stay dry and asks to keep using diapers. The mother is desperate and asks what she can do to get her son toilet trained immediately. Which of the following is the best advice to give the mother of this patient? A) To stop trying to toilet train for a few months then reassess B) To stop trying to toilet train for a year then reassess C) To continue to motivate the child and discipline him if he fails to void appropriately D) To obtain an evaluation from a developmental psychologist E) To stop using positive reinforcement rewards
A) To stop trying to toilet train for a few months then reassess Toilet training occurs between 2-4 years of age. If the child is not motivated to stay dry, the best advice is to wait for a few months then reassess. This will give the child and mother a break and allow for both to approach the situation fresh. Regardless, it is normal for children to not use the toilet up until age 5. Toilet training occurs between ages 2 and 4 with 5 being the cutoff age when an inability to use the toilet may be considered a problem. To be ready for toilet training, the child must be aware of bladder filling, able to consciously tighten their external sphincter, has normal bladder growth, and be motivated to stay dry. Further, the child must have met motor milestones and be able to walk to the toilet, sit upright, as well as have met language milestones to communicate the need to use the bathroom. The process involves placing the child on a toilet at regular intervals and using positive reinforcement as a reward. If the child is very reluctant, the best strategy is to stop trying to toilet train for a few months and then reassess.
Which of the following leads to retropatellar pain? A) Increased Q angle B) Increased quadriceps tone and strength C) Osgood-Schlatter Disease D) hamstring stretching
A) increased Q angle Patients with an increased Q angle have more force directed laterally during knee flexion and are at greater risk of patellofemoral pain syndrome.
A mother brings in her five year-old boy for his school physical. She voices some concerns about his readiness for school, saying he seems to be socially immature. She has noticed he does not interact with other children well, and that when he plays with them, he has a tendency to "place them" and then run around them as if they were statues. He rarely cries when he is hurt, and he shrugs off any attempt to hug him. He has good attention to details, and will sit and draw the same geometric shapes over and over again, but does not seem interested in learning the alphabet. He avoids eye contact with anyone. What is the most likely diagnosis?
Autism Children with autism do not tend to make eye contact, and even avoid it. They do not accept comfort when hurt and stiffen up when hugged. They do not tend to play with others, and do not tend to imitate grown-ups in play. They approach play in a more mechanical way, using others as props rather than interacting with them.
Examples of Social/Emotional Developmental Disorder
Autism spectrum disorders Reactive attachment disorder
A severely dehydrated child with gastroenteritis who is unable to tolerate oral rehydration should receive which of the following intravenous therapies? A) 10 mg/kg normal saline B) 20 mg/kg normal saline C) 10 mg/kg D5W D) 20 mg/kg D5W
B) 20 mg/kg normal saline In a severely dehydrated child, restoring intravascular volume to ensure adequate tissue perfusion is the immediate objective. This is best done with either Ringers lactate or normal saline. The addition of potassium would only be done after initial fluid boluses and after ensuring adequate kidney function. The addition of glucose to the IV solution may result in an osmotic diuresis worsening the dehydration.
Which of the following is the treatment of choice for a torus (buckle) fracture involving the distal radius? A) Open reduction and internal fixation B) Ace wrap or anterior splinting C) Closed reduction and casting D) Corticosteroid injection followed by splinting
B) Ace wrap or anterior splinting A torus or buckle fracture occurs after a minor fall on the hand. These fractures are very stable and are not as painful as unstable fractures. They heal uneventfully in 3-4 weeks
A 12-month-old boy presents with two days of nasal congestion, red eyes, and fever. On examination, he has bilateral purulent conjunctivitis and purulent nasal discharge. Both tympanic membranes are erythematous, bulging, and dull with limited mobility. What therapy is indicated? A) Amoxicillin B) Amoxicillin-clavulanate C) Cefdinir D) Otic ciprofloxacin drops
B) Amoxicillin-clavulanate (TOC for URI with bilateral acute otitis media)
An 8-year-old boy presents to his pediatrician's office with his mother after a recent parent-teacher conference. The teacher informed the mother that the boy is very disruptive in class, often leaving his seat or blurting out answers before being called. He is also impulsive and unable to wait for his turn. He often blames others for his mistakes and misbehaviors. As a result of this, he has few friends in his school and his academic performance has declined. At home, the mother reports that the child also has similar behaviors. He is spiteful and wants to get even with his sibling, he refuses to follow directions and constantly interrupts adult conversations. He is always "on the go" and runs about or climbs too much when remaining seated is expected. In the clinic, the child is restless and interrupts your conversation with the mother often. Which of the following is the most effective therapy for the child? A) Atomoxetine B) Dextroamphetamine C) Guanfacine D) Sertraline
B) Dextroamphetamine (stimulants TOC for ADHD)
A 6 year-old female presents to the clinic with her parents because of lower back pain and difficulty walking for one week. The pain is described as a dull ache that is constant in the midlumbar region with radiation into the abdomen. She has a low grade fever of 100 degrees F which the parents note has been persistent for several days. Upon exam it is noted that the child has restricted forward flexion and extension secondary to pain. She has tenderness over the L3 vertebrae. An area of cellulitis from an insect bite is discovered over her left scapula. Which of the following is the most likely diagnosis? A) Scheurmann's disease B) Discitis C) Lyme disease D) Osteitis condensans ilii
B) Discitis Discitis is an infectious disease that occurs in the mid lumbar region of children at the age of about 6 years old. Symptoms are low back pain that radiates into the abdomen or lower extremity. A low-grade fever with nausea and vomiting occasionally occur.
A 12-year-old boy presents with his mother for his annual physical exam The mother states she is concerned that her son is very short compared to the rest of his classmates, while just a few years ago his height seemed comparable to his peers. She says he sits for long periods of time, does not want to play sports, and tires easily. He complains of being cold all the time. On physical exam, a diffusely enlarged thyroid gland is noted. His serum thyroid stimulating hormone (TSH) and serum free thyroxine (T4) are tested. Which of the following lab results would indicate primary hypothyroidism as the cause of his symptoms? A) Elevated TSH and elevated T4 B) Elevated TSH and low T4 C) Low TSH and elevated T4 D) Low TSH and low T4
B) Elevated TSH and low T4 (indications of Hypothyroidism)
A three-week-old infant presents to the clinic with his parents who state he is not gaining weight and is feeding poorly. Cardiac auscultation and exam reveal a 2/6 mid-frequency holosystolic murmur heard best at the fourth left intercostal space, a diastolic rumble, and a prominent apical impulse displaced laterally. Tachypnea and tachycardia are also noted. Which of the following is the most likely diagnosis? A) Large atrial septal defect B) Large ventricular septal defect C) Patent ductus arteriosus D) Tetralogy of Fallot
B) Large ventricular septal defect
A mother of a newborn infant presents to the office concerned about reducing the risk of sudden infant death syndrome (SIDS). The infant was delivered at 39 weeks gestation weighing 7 pounds 9 ounces. There is no family history of SIDS and this is her first child. Which of the following is appropriate advice to reduce the risk of SIDS? A) Bottle feeding with soy formula B) Offer a pacifier at nap and bedtimes C) Have the infant sleep in the prone position D) Infant should sleep with the parents to allow close observation
B) Offer a pacifier at nap and bedtimes Use of a pacifier during sleeping is a current recommendation to decrease the risk of SIDS.
An 18-year-old pregnant woman presents to her primary care physician for treatment of her acne vulgaris. Which of the following would be the recommended treatment? A) Isotretinoin B) Oral erythromycin C) Oral tetracycline D) Topical tazarotene
B) Oral erythromycin (only option safe in pregnancy) Isotretinoin and topical Tazarotene are Class X. Oral Tetraycline is Class D.
A 4-year-old boy presents to the clinic with his very anxious mother. She reports that her son's penis does not look normal and he complained of pain when she tried to examine it at home. She denies history of abdominal pain or vomiting. Past medical history is unremarkable. On physical examination, the foreskin appears retracted and edematous and cannot be pulled back over the glans. What is this patient's most likely condition? A) Hypospadia B) Paraphimosis C) Phimosis D) Priapism
B) Paraphimosis
A 3-week-old girl is referred to the ED by her pediatrician who has been following her closely due to herpes simplex exposure via primary maternal infection. The patient's mother insists she had no active lesions during pregnancy. In the last 12 hours, the infant developed a fever of 103°F and parents also report two brief seizure-like episodes, lethargy, and poor feeding. On physical exam, the infant is sleepy but arousable. A full septic workup is initiated, including a lumbar puncture. Which of the following tests of the patient's cerebrospinal fluid is most likely to confirm the diagnosis? A) Gram stain B) Polymerase chain reaction C) Serology D) Viral culture
B) Polymerase chain reaction (TOC for HSV)
A mother's concern about her child's choppy, gasping snore prompts the practitioner to suspect the possibility of which of the following as the cause of respiratory obstruction? A) inclusion cysts B) enlarged adenoids C) high-arched palate D) geographic tongue
B) enlarged adenoids Large adenoids are considered part of the differential diagnosis of snoring and upper airway obstruction.
A 15 year-old male presents with a 1 week history of hacking non-productive cough, low grade fever, malaise and myalgias. Examination is unremarkable except for a few scattered rhonchi and rales upon auscultation of the chest. The chest x-ray reveals interstitial infiltrates and a cold agglutinin titer was negative. Which of the following is the most likely diagnosis? A) acute bronchitis B) viral pneumonia C) mycoplasma pneumonia D) pneumococcal pneumonia
B) viral pneumonia The patient's clinical symptoms as well as chest x-ray findings and negative cold agglutinin titer are most consistent with viral pneumonia.
While commonly found in adults with rheumatoid arthritis, which of the following is only rarely found in a patient with juvenile idiopathic arthritis? A) Antinuclear antibodies B) Human leukocyte antigens C) Rheumatoid factors D) Tumor necrosis factor
C) Rheumatoid factors
A 10 year-old female experiences fever and polyarthralgia. On examination you note a new early diastolic murmur. Laboratory results are positive for antistreptolysin O. The patient has no known drug allergies. Which of the following is the recommended prophylaxis for this condition? A) pneumonia B) bronchiolitis C) croup D) asthma
Benzathine penicillin G Recurrences of rheumatic fever are most common in patients who have had carditis during their initial episode and in children. The preferred method of prophylaxis is Benzathine penicillin G every four weeks.
A 4-year-old girl presents with a three day history of malaise, fever of 40o C, cough and coryza. She then developed an erythematous maculopapular rash that started on the face before spreading to the trunk and lower body. Her immunization record shows that aside from an initial dose of hepatitis B vaccine given at delivery, no other vaccinations have been given. On physical examination, the patient is weak-looking and febrile with a generalized maculopapular rash, watery nasal discharge, white pinpoint lesions on bright red mucosa opposite the lower molars, and occasional crackles upon auscultation. Which of the following is the most likely diagnosis? A) Fifth disease B) Rubella C) Rubeola D) Sixth disease
C) Rubeola
A four-year-old boy is brought to your office by his concerned mother who states that he intermittently turns blue. She reports that he has been growing and developing normally, but recently he has been having crying fits where he turns blue and then squats down into a ball. Vital signs are stable. Physical examination reveals a pansystolic murmur at the left lower sternal border. Which of the following is true regarding the disease affecting this patient? A) The condition is associated with Turner's syndrome B) The condition is the MC congenital cardiac anomaly C) A boot-shaped heart will be found on chest radiograph D) The condition may be treated with Indomethacin E) A chest radiography would reveal notching of the ribs
C) A boot-shaped heart will be found on chest radiograph This patient has cyanotic spells, relieved by squatting, consistent with Tetrology of Fallot (TOF). One can expect a boot-shaped deformity on chest radiograph due to right ventricular hypertrophy. In TOF, patients have the classic clinical features of pulmonary stenosis (the most important determinant for prognosis), right ventricular (RV) hypertrophy, and an (overriding) aorta that lies directly over a ventricular septal defect (VSD). Because of pulmonary stenosis (elevated RV pressures) and the overriding aorta, patients experience early cyanosis from right to left intracardiac shunting. Patients may squat to improve symptoms because compressing the femoral arteries increases total peripheral resistance and decreases the right to left shunt, directing more blood from the right ventricle to the lungs and less across the VSD and into the aorta.
A 16 year-old male is brought into your office by his girlfriend. She states that "he hasn't been himself lately" and seems to fluctuate from being almost "euphoric" to being depressed and irritable. The patient states that "he is really okay" and that he "just feels a little irritable occasionally." On physical examination his pulse is 120 beats/minute, blood pressure is 180/110 mmHg, he is sweating and his pupils are widely dilated. Which of the following is the most likely diagnosis? A) Opiate abuse B) Acute anxiety attack C) Cocaine intoxication D) Bipolar affective disorder
C) Cocaine intoxication Cocaine is a stimulant and presenting clinical manifestations of intoxication include agitation, tachycardia, hypertension, diaphoresis, and dilated pupils.
A 2 year-old child is brought to the office because of a cough and a fever of 102 degrees F for 2 days. The physician assistant notes the presence of hoarseness, a barking cough, and stridor. The ears and nose exam are unremarkable. Auscultation of the chest reveals decreased breath sounds without crackles or expiratory wheezes. Which of the following would be the initial diagnostic impression? A) pneumonia B) bronchiolitis C) croup D) asthma
C) Croup Hoarseness, inspiratory stridor, and a barking cough are classic signs of croup, all of which are noted in this patient.
A 4-month-old female presents with her parents to your office. The patient's parents are concerned because she used to push up to her elbows during tummy time, but she now struggles to lift her head. She has been urinating and stooling well, but her parents report decreased interest in food. All prenatal screening for the patient was unremarkable. There is no family history of genetic abnormalities in the patient's mother. The patient's father was adopted and knows little about his biological family. The patient has two older siblings who are both developmentally normal. On physical exam, the patient appears well-developed and well-nourished. She is in the 40th and 48th percentiles for height and weight, respectively. She has no dysmorphic features. Her abdomen is soft and non-tender. Hepatosplenomegaly is present. Ophthalmologic exam reveals the finding seen here. Of the following, which is most likely to be found in this patient? A) Corneal clouding B) Hypohydrosis C) Hyporeflexia D) Hyperreflexia
C) Hyporeflexia This 4-month-old patient presents with loss of motor milestones, a "cherry-red" spot on the macula, and hepatosplenomegaly, which suggests a diagnosis of Niemann-Pick disease. Niemann-Pick disease also presents with hyporeflexia. Niemann-Pick disease (NPD) is a sphingolipidosis caused by a deficiency in sphingomyelinase. NPD shares many features with Tay-Sachs disease, as both present in infants aged 2-6 months with progressive neurodegeneration, hypotonia, feeding difficulties, and a "cherry-red" macula. The two can be distinguished by the fact that only NPD presents with hepatosplenomegaly, and NPD presents with hyporeflexia or areflexia, whereas Tay-Sachs disease presents with hyperreflexia. In addition, NPD is characterized by an accumulation of sphingomyelin and other lipids in macrophages, which gives affected cells a "foamy" appearance.
An 18-month-old boy presents with a one-week history of upper respiratory congestion and low-grade fever. His parents report noisy breathing increasing over the last 24 hours, and a loud "seal-like" barking cough. He has no significant past medical history. Which of the following, if reported in the history given by parents, is most consistent with your suspected diagnosis? A) Drooling B) Expiratory wheeze C) Inspiratory stridor D) "Hot-potato", muffled voice
C) Inspiratory stridor (hallmark sign of croup)
A 2-year-old girl presents with six days of fever. She has also had a mild cough, red eyes, and nasal congestion. Vital signs are notable for a temperature of 102.5°F and heart rate 135 beats per minute. Examination findings include bilateral conjunctival injection, rhinorrhea, red tongue, bilateral shotty anterior cervical adenopathy with a prominent right-sided lymph node, and swollen hands with erythematous palms. Which of the following is most likely to resolve her symptoms? A) Amoxicillin B) Honey, acetaminophen, and a cool mist humidifier C) Intravenous immunoglobulin D) Vancomycin
C) Intravenous immunoglobulin (TOC for Kawaski's disease)
A 15-year-old girl presents to your clinic for evaluation of short stature and academic difficulties. She is a sophomore in high school and has always struggled in mathematics. Previous cognitive testing showed a full scale intelligence quotient of 94, with her verbal skills significantly higher than her non-verbal skills. She has been teased quite a lot for her height as she is the smallest in her class. Past medical history is otherwise unremarkable. The patient is in the 50th percentile for weight but is less than 5th percentile for height. On physical examination, pertinent findings include a short, webbed neck and delayed breast development. The patient also reports that she has not started menstruating yet. What is the most appropriate test to order to confirm the likely diagnosis? A) Buccal smear for Barr bodies B) Follicle stimulating hormone and luteinizing hormone levels C) Karyotype analysis D) Serum 17-hydroxyprogesterone level
C) Karyotype analysis (DOC for Turner's syndrome)
A 10-year-old girl presents to clinic due to a generalized rash. History reveals that four days prior the onset of rash she was noted to have low-grade fever, runny nose, cough, and conjunctival congestion. The rash is characterized as red, maculopapular eruptions which were noted to first appear on the forehead before spreading downward to the extremities. The physical examination reveals a weak-looking girl with anicteric sclerae, pink conjunctivae, cervical and occipital lymphadenopathy, harsh breath sounds, adynamic precordium, no heart murmur, soft non-tender abdomen, full equal pulses, capillary return time of less than two seconds, and maculopapular rashes on the lower trunk and extremities with brawny desquamation on the upper trunk. Further history reveals that she is missing some of her immunizations. Which of the following is the most likely diagnosis? A) Exanthem subitum B) Kawasaki disease C) Measles D) Rubella
C) Measles
A 2-month-old boy presents to the emergency department with his mother after she noticed that he turned blue during his last feeding. In the emergency department, the baby had a heart rate 135 beats per minute, respiratory rate 38 breaths per minute and an oxygen saturation of 84% on room air. On physical examination, the baby was crying and appeared cyanotic. You were able to console the baby briefly with a pacifier. Upon auscultation, you appreciated a grade 3/6 harsh, systolic ejection murmur loudest at the left upper sternal border. The baby began crying again and became more cyanotic. Which of the following is the most appropriate next step in the management? A) Administer oxygen and aim for oxygen saturation of 95% or greater B) Administer prostaglandin C) Place the baby in a knee-to-chest position D) Recommend urgent surgical correction
C) Place the baby in a knee-to-chest position -best maneuver for Tetralogy of Fallot symptom relief
Which of the following increases the risk of sudden infant death syndrome? A) Pacifier use B) Immunizations C) Prone sleeping D) URI
C) Prone sleeping The risk of sudden infant death syndrome is increased with infants sleeping prone, exposure to cigarette smoking, low birth weight, and preterm birth
A 5-week-old boy presents with his father to the emergency room because of a ten day history of vomiting that has become more frequent and more forceful. The vomitus is not bile stained. The infant sucks vigorously and is active, but he is observed to be losing weight. There are no other symptoms noted. Abdominal examination reveals a small, olive-shaped mass upon palpation. Which of the following is the most likely diagnosis? A) Intussusception B) Malrotation C) Pyloric stenosis D) Volvulus
C) Pyloric stenosis
A 10-year-old boy with a history of sickle cell disease presents to the primary care office for his annual physical. His mother states that he has been complaining of muscular pain and loss of appetite; he refuses to eat any sort of vegetable or fruit. She has noticed that he bruises easily and when he has a wound it takes a long time to heal. At his last dental visit he had bleeding gums and several cavities. His plasma concentration of ascorbic acid is less than 0.2 mg/dL. What is the most appropriate treatment at this time? A) Ferrous sulfate 3 mg/kg once daily between meals B) Iron-fortified multivitamin once daily between meals C) Vitamin C 100 mg three times daily for one week, then once daily D) Vitamin C 1000 mg daily for one month
C) Vitamin C 100 mg three times daily for one week, then once daily (TOC for Vitamin C deficiency correction)
A 3-day-old term infant presents to the pediatrician for his first health check. There were no complications during delivery, and the infant is breastfeeding well per his mother. She eats a normal diet, has no significant past medical history, and plans to breastfeed exclusively for the first six months of his life. Which of the following supplements is recommended for the infant? A) Vitamin A B) Vitamin B12 C) Vitamin D D) Vitamin K
C) Vitamin D -low in all breastmilk
A 19 year-old woman has been consuming up to six beers daily since she was 16. She is now pregnant with her first child, has had little prenatal care, and is due to deliver in four weeks. Of the following, which neonatal problems should you anticipate? A) kernicterus B) hydrocephalus C) low birth weight D) teeth discoloration
C) low birth weight Fetal alcohol syndrome is a common cause of low birth weight.
Which of the following is an indication for hospitalization in a patient who has acute bronchiolitis? A) pulse oximetry of 94% on room air B) children between 4-6 months of age C) moderate tachypnea with feeding difficulties D) hyperinflation and interstitial infiltrates on chest x-ray
C) moderate tachypnea with feeding difficulties Indications for hospitalization include moderate tachypnea with feeding difficulties.
Human Papilloma Virus (HPV) Vaccine Timing
Can be given anytime from 9 - 41 years (was 26 yrs) If <15 years: 2 doses 6 months apart If 15-41 years: 3 doses with 6 months between the 1st and 3rd (aka 1st dose-2 mos-2nd-4 mos-3rd dose)
A 6-year-old child with sickle cell anemia presents with fever and pain over the right tibia. There is tenderness along the anterior tibia, but no pain with motion of the knee or ankle. What is the most appropriate initial treatment?
Cefotaxime Cefotaxime provides coverage for osteomyelitis caused by staphylococcus or salmonella.
Examples of Gross Motor Developmental Disorder
Cerebral palsy Muscular dystrophies DCD
Pubic Hair Tanner Stage 4
Characterized by pubic hair that resembles adult pubic hair, although the escutcheon covers a smaller area than seen in adults
9-12 Year EPSDT
Check fasting labs: CBC, CMP, T4, TSH, Lipid (annually if overweight + RD, code under overweight if AHCCCS or WCC if private) Depression screen with PHQ-9 at 12 yrs Review recommended vaccines Psychosocial assessment(bullying, development, school issues)
A 2 year-old presents to the emergency department in acute respiratory distress. The parents relate a history of a recent upper respiratory illness that was followed by a sudden onset of barking cough during the night, but this morning they noted increased difficulty breathing. The child is noted to have stridor at rest, but has no evidence of cyanosis. What is the most appropriate initial intervention?
Nebulized racemic epinephrine This patient most likely has laryngotracheobronchitis (viral croup). Treatment with nebulized racemic epinephrine and glucocorticosteroids is indicated for patients with stridor at rest.
18 Month EPSDT
Confirm pt off formula, bottles, pacifiers Review development (walking/running, climbing stairs, saying 10-15 words, follows commands, potty-training) Review MCHAT and scoring Confirm previous CBC/Lead were performed Refer to AZEIP if not meeting milestones Review recommended vaccines
A 6 month old male presents with a scrotal mass. The scrotum is swollen and testicles are non-tender. The scrotum does transilluminate. What is the treatment of choice?
No treatment is needed at this time A hydrocele presents with a swollen, non-tender scrotum. Due to the scrotum being filled with fluid, the mass will transilluminate. No treatment is required unless the hydrocele persists after the age of 2 years.
A mother brings her 14 month-old son to your clinic. Earlier today she lifted her son by grabbing him by the wrists and pulling him up off the floor. The child is sitting in his mother's lap with his left forearm is extended and in pronation. He is refusing to move the left arm, forearm or wrist. The arm and joints appear normal with no noted deformities, edema or erythema. Distal pulses and capillary refill are normal and he can move his fingers. What is the most likely diagnosis?
Nursemaid's elbow This clinical history is classic for radial head dislocation or nursemaid's elbow.
A 6 year-old boy is brought to the pediatric clinic by his mother for an evaluation of his asthma. He coughs about 3 days out of the week with at least 2-3 nights of coughing. Which of the following would be the most appropriate treatment for this patient? A) Mast cell stabilizer B) Long acting beta agonist C) Leukotriene receptor antagonist D) Low dose inhaled corticosteroid
D) Low dose inhaled corticosteroid Low dose inhaled corticosteroids are the preferred treatment for mild persistent asthma.
A 10-year-old boy is brought to the outpatient office for follow-up. He was recently diagnosed with attention-deficit/hyperactivity disorder. What is the treatment of choice for this patient? A) Atypical antidepressants B) Behavior modification therapy alone C) Selective serotonin reuptake inhibitors D) Stimulants
D) Stimulants (TOC for ADHD)
A 13-year-old boy presents with four days of sore throat, low grade fever, and generalized malaise. He denies significant nasal congestion or cough. On examination, he has 2+ erythematous tonsils covered in white exudate, bilateral posterior cervical adenopathy, as well as shotty axillary and inguinal lymphadenopathy. Which of the following is indicated? A) Course of oral prednisone B) Drainage of peritonsillar abscess C) Intramuscular penicillin G D) Supportive care
D) Supportive care (TOC for mononucleosis)
A 16-year-old boy presents for his annual physical. He denies any complaints. Baseline labs show an elevated calcium level and low vitamin D. Additional testing confirms an elevated parathyroid hormone level. Surgery has been declined at this time. Which of the following should be avoided as it has been shown to exacerbate hypercalcemia? A) Bisphosphonates B) Calcimimetic agents C) Moderate vitamin D intake D) Thiazide diuretics
D) Thiazide diuretics Patient presentation: asymptomatic primary hyperparathyroidism (common cause of hypercalcemia) *avoid Thiazides and Lithium with hypercalcemia*
A 10-year-old boy with a history of mild persistent asthma is seen by his pediatrician for refill of his asthma medications. He carries his own albuterol inhaler and has been using it more often than expected. He does have some controlled eczema but is otherwise healthy. Before providing the script for a refill, his other medications are reviewed and adherence is discussed with the patient. The patient is asked to describe his inhaler technique and screened for any recent illnesses which may be exacerbating his symptoms. What is the most important non-pharmacologic component of management to discuss with the patient and family? A) Chest physiotherapy B) Exercise C) Incentive spirometry D) Trigger avoidance
D) Trigger avoidance
What is the most important measure in the prevention of epiglottitis?
Haemophilus influenzae B vaccine Immunization with the HIB vaccine has greatly reduced Haemophilus influenzae as a cause of epiglottitis.
A 3 year-old child playing in an abandoned shed is bitten by a black widow spider. The mother rushes the child to the emergency department within 20 minutes of the incident. Which of the following if the best initial intervention? A) IM steroids B) Administration of antivenin C) Immediate immersion in a cold bath D) Hospital admission for symptomatic care
D) Hospital admission for symptomatic care Hospital admission for symptomatic care should be considered in children, pregnant women, and patients with preexisting cardiovascular disease.
A 37-year-old-woman delivers a male infant with Down syndrome at 38 weeks. His Apgar scores at one and five minutes are both 8, with scores of 1 for acrocyanosis and irregular breathing. He passes meconium several hours later. Between 24 and 48 hours after birth, the infant has multiple episodes of vomiting despite lack of feedings and does not have additional bowel movements. Vomitus is bilious and nonbloody. On physical exam, the infant is crying, he appears jaundiced, and his abdomen is distended. An abdominal radiograph is performed, which is shown here. Which is the first step in the management of this patient? A) initiation of phototherapy B) abdominal ultrasound C) endotracheal intubation D) nasogastric tube placement E) immediate surgical intervention
D) Nasogastric tube placement The clinical presentation and radiograph are consistent with duodenal atresia. Nasogastric or orogastric tube placement is the first step in management prior to surgical intervention. Duodenal atresia is the congenital failure of the duodenal lumen to recanalize during fetal development. The condition is often seen in infants with Down syndrome and is associated with a number of congenital anomalies including biliary atresia, as well as cardiac and renal malformations. Duodenal atresia presents with vomiting 1-2 days after birth. Abdominal radiograph reveals the "double bubble" sign, caused by gastric and proximal duodenal dilation with narrowing at the pylorus.
A 16-year-old boy presents with mild left-sided hearing loss following a scuba diving trip. He also noticed reddish discharge from the left ear but denies vertigo or facial muscle weakness. On physical exam, there is a perforation of his left tympanic membrane. Which of the following is the appropriate management of this patient? A) Aural toilet B) Follow-up in eight weeks C) Gentamicin otic drops D) Ofloxacin otic drops
D) Ofloxacin otic drops (TOC for perforated tympanic membrane with minimal hearing loss and no vestibular symptoms)
A 13 year-old male with known cystic fibrosis presents to the emergency department, accompanied by his parents, with increased coughing, wheezing and low grade fever. Rales are audible on auscultation of the lungs. Treatment should target which of the following organisms? A) Pneumocystis jiroveci B) Haemophilus influenze C) Mycoplasma pneumoniae D) Pseudomonas aeruginosa
D) Pseudomonas aeruginosa Pseudomonas aeruginosa is the most predominant pathogen in patients with cystic fibrosis.
A 3-year-old girl with no significant past medical history presents to her pediatrician. She wet her bed overnight for the first time since being toilet-trained and had strong-smelling urine. Today, she had a low-grade fever of 100.4°F. Her mother reports she has not eaten much today but is drinking and voiding frequently. She did have some mild diarrhea recently but no vomiting. A urine dipstick is sent and notable for a cloudy appearance with +2 bacteria. Leukocyte esterase and nitrites are both positive. The patient has previously developed urticaria when taking amoxicillin; no follow-up skin testing has been done yet. Local antibiograms show no patterns of resistance which would exclude antibiotics active against common pathogens. Which of the following is the most appropriate therapy for this patient? A) Cefixime B) Ciprofloxacin C) Nitrofurantoin D) Trimethoprim-sulfamethoxazole
D) Trimethoprim-sulfamethoxazole (TOC for uncomplicated pediatric UTI)
A 2-year-old boy presents to the ED with a three-day history of increasing fussiness, decreased appetite, and intermittent non-bloody, non-bilious vomiting. He has not had any sick contacts, but he recently visited relatives who live in an old home currently undergoing renovations. His mother is concerned he may have been exposed to lead paint. Which of the following is the best test to confirm the diagnosis? A) Bone lead level B) Capillary blood lead level C) Dentine lead level D) Venous blood lead level
D) Venous blood lead level (gold standard for diagnosis)
Which of the following should be recommended to parents of a child with pediculosis capitis? A) Avoid skin-to-skin contact with the patient B) Clean surfaces with bleach C) Keep the child out of school until medicated for at least 24 hours D) Wash clothing and linens in hot water
D) Wash clothing and linens in hot water Pediculosis capitis = lice
A 4-year old boy presents to the office for his well-child examination. Upon listening to his heart, you hear a soft murmur. Which of the following features would be concerning for a pathologic murmur? A) A crescendo/decrescendo quality B) Split S2 sound that changes with inspiration C) Systolic murmur that decreases in intensity with standing D) Widely split and fixed S2
D) Widely split and fixed S2 -indicative of an atrial septal defect instead of physiologic murmur
Which of the following clinical manifestations is most commonly seen in viral croup? A) Drooling B) Wheezing C) Sputum production D) Inspiratory stridor
D) inspiratory stridor Viral croup typically presents with a barking cough and stridor.
Examples of Fine Motor Developmental Disorder
DCD Dysgraphia
A 7 year-old boy wets the bed nearly every night. What is the best pharmaceutical agent to use in treating this patient?
Desmopressin (DDAVP) Desmopressin, while not curative, will relieve symptoms.
A 1-day-old develops bilious vomiting without abdominal distension. Abdominal xray reveals a double-bubble sign. What is the intervention of choice for this patient?
Duodenoduodenostomy Duodenal atresia presents within the first day of life with bilious vomiting without abdominal distention . A double-bubble sign is noted on abdominal xray film. Treatment of choice is a duodenoduodenostomy.
A 3-day-old male infant born at 39 3/7 weeks to a 29-year-old G2P1001 mother exhibits yellow discoloration of the skin notable from the forehead to the level of the umbilicus. Which of the following would be concerning and prompt further evaluation of this infant? A) Total serum bilirubin of 10 mg/dL at 72 hours of life B) Conjugated serum bilirubin of 1.6 mg/dL at 60 hours of life C) Appearance of the infant's jaundice first occurring at 48 hours of life D) Clinical persistence of the infant's jaundice through 1 week of age E) Total serum bilirubin of 4 mg/dL at 48 hours followed by 11 mg/dL at 72 hours
E) Total serum bilirubin of 4 mg/dL at 48 hours followed by 11 mg/dL at 72 hours This infant's clinical picture is consistent with a diagnosis of neonatal jaundice. A rising serum total bilirubin at a rate of greater than 5 mg/dL/24hr is indicative of pathologic jaundice and warrants further investigation. It is important to differentiate pathologic from physiologic neonatal jaundice, as there are numerous potential underlying causes to pathologic hyperbilirubinemia that, if present, must be identified so that further work-up and treatment can be initiated. Indications to pursue further evaluation of neonatal jaundice include: 1) conjugated bilirubin > 2 mg/dL or > 20% of total bilirubin, 2) jaundice in the first 24 to 36 hours, 3) bilirubin rate of rise > 5 mg/dL/24hrs, 4) total bilirubin > 12 mg/dL in a term infant, 5) jaundice persisting beyond 10-14 days of life, and 6) presence of any additional signs or symptoms indicative of underlying disease.
EPSDT Exam
Early Periodic Screening Diagnosis and Treatment
A 3-month-old female is brought by her mom for a routine physical examination. The patient's mom has no complaints. On examination you note a well-developed, well-nourished infant in no apparent distress. There is no cyanosis noted. Heart examination reveals a normal S1 with a physiologically split S2. There is a grade III/VI high-pitched, harsh, pansystolic murmur heard best at the 3rd and 4th left intercostal spaces with radiation across the precordium. What is the initial diagnostic study of choice in this patient?
Echocardiogram An echocardiogram is the initial diagnostic study of choice in the diagnosis of a VSD.
A 23 month-old male presents to the office with his father who reports the patient has had an acute onset of severe pharyngitis, fever of 103.5 degrees F and what sounds like harsh, high-pitched breath sounds. His dad states the child has started drooling and seems to be worsening. The child is not presently crying but has muffled voice sounds. The child has not been immunized due to religious reasons. What is the most appropriate next step?
Emergent transfer Epiglottitis requires endotracheal intubation to maintain the airway but should be performed only in the operating room or emergency room with a competent physician prepared to place an endotracheal tube or less often to perform a tracheostomy.
A 2 month-old infant has been diagnosed with pneumonia due to Chlamydia trachomatis. What is the treatment of choice?
Erythromycin or sulfisoxazole is the treatment of choice for an infant with Chlamydial pneumonia.
Functions Affected by Adult Loss of Fine Motor Development
Feeding Dressing Elimination (toileting
Developmental Milestones at 5 yrs
G: Balance on one foot for 10 seconds, skips, may learn to ride a bicycle F: Draw person, knows 10 body parts, tripod pencil grasp, prints name, copies letters, independent ADLs including tying S: 5000 words, future tense, word play/jokes/puns, phonemic awareness C: Counts to 10 accurately, recite ABC's, recognizes some letters, pre-literacy and numeracy skills S: Has group of friends, follows group rules, games with rules
Developmental Milestones at 2 mos
G: Head steady when held, head up to 45 prone F: Hands open half the time, bats at objects S: Turns to voice, cooing C: Prefers usual caregiver, attends to moderate novelty, follows past midline S: Attachment, social smile
HEEADSSS Exam
Home environment Education and employment Eating Activities (peer-related) Drugs Sexuality Suicide/depression Safety (from injury and violence)
A 3 month-old male presents with a hoarse cough and thick purulent rhinorrhea for the past 2 days. The mother noted that yesterday he appeared to get worse and seemed to have increasing problems breathing and trouble feeding. Examination reveals a temperature of 100.2 degrees F and respiratory rate of 80/minute with nasal flaring and retractions. Lung examination reveals a prolonged expiratory phase with inspiratory rales. He is tachycardic. Pulse oximetry reveals oxygen saturation of 89%. Chest x-ray reveals hyperinflation with diffuse interstitial infiltrates. What is the most appropriate intervention?
Hospitalization This infant most likely has bronchiolitis. While most cases are mild and can be treated at home, hospitalization is recommended for infants with hypoxia on room air, moderate tachypnea with feeding difficulties and marked respiratory distress with retractions. Additionally hospitalization is recommended for infants less than 2-3 months of age, a history of apnea or an underlying chronic cardiopulmonary disease.
A woman brings her 3 month-old son to the clinic. Upon examination, it is noted he has a round face, a large protruding tongue, dry skin, an umbilical hernia, and his weight gain is below average. He appears apathetic and the mother says the infant is usually constipated. What is the most likely diagnosis?
Hypothyroidism Congential hypothyroidism presents gradually, and at 3-6 months findings include poor appetite and feeding, sluggishness, constipation, enlarged abdomen and umbilical hernia, enlarged tongue, and the child does not meet developmental milestones.
An 18 month old presents with abdominal pain and bloody diarrhea. On physical examination a sausage shaped mass is noted in the upper mid-abdomen. What is most likely the diagnosis?
Intussusception Intussusception, telescoping of proximal bowel into distal bowel, is most common in children younger than age 2, who present with abdominal pain and bloody "currant" jelly" stool. On physical examination a sausage-shaped mass is noted in the mid abdomen. Patients with pyloric stenosis presents with vomiting and an olive size mass in the mid-abdomen. Those with Hirschsprung's disease present with vomiting and abdominal distention. Duodenal atresia is noted in the first day of life, with vomiting without abdominal distension
A 16-year-old male presents with decreased body hair and gynecomastia. On physical exam, the patient is noted to be tall and thin, with a wide arm span. Genital exam reveals a small phallus and small, soft testicles. What is the most likely the diagnosis?
Klinefelter's syndrome The Klinefelter syndrome is an autosomal recessive disorder due to an extra X chromosome. Manifestations typically present at puberty with incomplete masculinization, decreased the body hair, gynecomastia, small phallus, and small, soft testicles. Noonan's syndrome presents with short stature, web neck, and pectus excavatum. Turner's syndrome presents with short stature, webbed neck , and phenotypical female. Trisomy 18 presents with mental retardation, prominent occiput and low-set ears
A 9 year-old male is brought in by his mother who reports the patient has exhibited an extremely negative attitude for the past year. He seems angry much of the time and frequently loses his temper. Arguing over even trivial details is common place and he seems to take delight in annoying his family. His grades and conduct at school remain excellent. He has few friends, though he has never been seen bullying or destroying others' property. What is the most likely diagnosis?
Oppositional defiant disorder Oppositional defiant disorder (ODD) best fits this scenario and is differentiated from conduct disorder by the lack of bullying and the lack of destruction of property. Many children with ODD do drift into conduct disorders over time. His good grades and conduct at school lessen the probability of untreated ADD. Personality disorders (i.e. antisocial personality disorder) can not be diagnosed at this early an age.
A 3 year-old presents with profuse watery diarrhea for the past three days. The child vomited twice yesterday, but not today. On exam, the child is febrile, with pulse of 142, respiratory rate of 18, and blood pressure of 60/40 mmHg. On exam, the child is alert and responsive, with no focal findings. What is the most appropriate intervention?
Oral rehydration The goal of therapy for a child with severe gastroenteritis and dehydration is to restore fluid loss. Oral rehydration with an appropriate electrolyte solution is the best option if the child is not actively vomiting and is alert enough to take oral fluids. IV fluids should be reserved for those who are unable to take fluids orally.
A 3-year-old presents with sore throat and fever. The child appears toxic and is having trouble handling oral secretions. What is the next most appropriate step in the evaluation of this patient?
Order a lateral neck x-ray study. In a patient with suspected epiglottis, lateral neck x-ray films may be helpful in demonstrating a classic "thumbprint" sign that will guide intervention
What would be expected on physical examination of a newborn diagnosed with Tetrology of Fallot?
Palpable right ventricular lift Tetralogy of Fallot is commonly associated with a palpable right ventricular lift.
Examples of Adult Functional Loss of Gross Motor Development
Paresis/paralysis
Examples of Adult Functional Loss of Fine Motor Development
Paresis/paralysis Apraxia
A 5 year-old male is brought to the office by his father who reports a 2 day history of low grade fever and coryza. The child awoke this morning with bright red cheeks. Physical examination reveals edematous, confluent plaques over the malar region of the face and reticular rash over the child's extensor surfaces. What is the most likely diagnosis?
Parvovirus B19 This is a classic presentation of Fifth Disease, a childhood exantham associated with human parvovirus B19.
A newborn is seen for an initial two week visit. Physical examination reveals a thrill and a continuous machinery murmur in the left second intercostal space. What is the most likely diagnosis?
Patent ductus arteriosus Patent ductus arteriosus is characterized by a classic harsh, machinery-like murmur that is continuous through systole and diastole. This is heard best at the left second interspace and is commonly associated with a thrill.
A 4-year-old patient presents with a sore throat and fever. Throat culture shows Group A beta-hemolytic streptococcus. What is the treatment of choice?
Penicillin Penicillin VK 250 mg po qid for 10 days is the recommended treatment for streptococcal pharyngitis. Erythromycin is an alternative in patients allergic to penicillin.
Red Flags in Gross Motor Development
Persistent primitive reflexes Abnormal tone or movement patterns at any age (spasticity, hypotonia, absent DTRs) Asymmetry Poor head control at 5 mos Not sitting independently with hands free at 8 mos Not rolling back-front, not taking weight well through the legs when held at 9 mos Not walking by 18 mos
Trends in Speech/Language Development
Primitive reflexes to volitional movement (suck) Proximal to distal (back of throat to lips, tongues, teeth) Understanding before speaking Increasing length of utterance Increasing complexity of syntax "Metas"
What is the most common pathogen implicated as the cause of bronchiolitis in children?
Respiratory syncytial virus
Newborn EPSDT
Review birth history + Mom's prenatal labs Pulse ox on right hand and foot (if not done at hospital) Compare current weight and birth weight (10% <) Encourage breastfeeding, WIC referral, formula Back to sleep, fever, jaundice, umbilical cord care Consult BH and RD for maternal screening Newborn labs Schedule 3 wk and 6 wk checks Begin ROAR program
9 Month EPSDT
Review development (crawling, pulling to stand, cruising, separation anxiety, making sounds, saying "mama"/"dada", imitates sounds) No scheduled vaccines (catch-up prn) Developmental screen (Peds tool) Dental referral Introduce sippy cups/review teething (no Orajel) Verbal Lead Screen
During an influenza epidemic, a 6 year-old child is seen with fever and a severe sore throat. A throat swab is taken for culture and the child is sent home. The next day, he is reported to have persistent vomiting and increased lethargy. On examination, he is delirious and disoriented. No rash is noted. His reflexes are hyperactive. The liver edge is 3 cm below the right costal margin in the midclavicular line. What is the most likely diagnosis?
Reye syndrome Reye syndrome is typically post-influenza or URI. The patient develops lethargy, drowsiness, and vomiting. Babinski reflex is positive and hyperreflexia is noted. The liver is normal or enlarged.
A 7-year-old patient presents with an abrupt onset of fever, chills, malaise, muscular aching, headache, and nasal stuffiness. The patient is taking two baby aspirin every 6 hours for fever, muscle aches, and headache. What complication is the patient most at risk for?
Reye's syndrome Reye's syndrome is a rare and severe complication of influenza, particularly in young children. The pathogenesis is unknown, but the syndrome is associated with aspirin use in viral infections.
Supplements for Headache Management
Riboflavin (B2) Magnesium (S/E: diarrhea, good and bad thing) CoQ10 Melatonin Omega 3 Fatty Acids
A 5 year-old child has just been hospitalized with meningococcemia. Family members and close contacts should be given which of the following as prophylaxis?
Rifampin orally Exposed household, school, or day-care contacts of children with meningococcemia should receive chemoprophylaxis to eliminate the organism from the nasopharynx. The drug of choice is rifampin (10 mg/kg, up to 600 mg) every 12 hours for 48 hours. Sulfisoxazole, ceftriaxone, and ciprofloxacin are alternative choices for prophylaxis.
A 14-year-old male is brought to an endocrinologist because his parents are concerned he is "too short." His pediatrician is contacted for his medical records, and his growth chart is obtained. If this patient's short stature is secondary to constitutional growth delay, what pattern is his growth curve most likely to follow?
Short stature but attaining a normal height by 18 years. The most common cause of short stature and pubertal delay is constitutional growth delay. This condition has a classic presentation on growth charts: normal birth weight and height, drop in percentiles on growth curve between 6 months and 3 years, re-establishment of normal growth velocity, and following the growth curve at the 5th to 10th percentiles. Adolescents will have a normal growth spurt and normal adult height. Constitutional growth delay is characterized by a bone age that is less than chronological age; the child's short height is often appropriate when assessed in the context of their skeletal age. The only management is reassurance and regular follow-up.
A mother brings a 3 month-old infant to the office because she is concerned about a red, vascular, nodular growth on the child's back. It appears to be enlarging slightly and the vessels are slightly dilated. It seems to cause the child no discomfort. The most likely diagnosis is
a hemangioma. A hemangioma is a bright red to deep purple vascular nodule or plaque that often develops at birth, may enlarge, and may regress and disappear with aging.
Rates of alcohol use in the adolescent are reportedly higher in:
adoptive children whose biological parent is an alcoholic There is a three-to fourfold increase in risk for alcohol dependence in adopted children whose biological parents are alcohol dependents.
A 6-year-old boy presents to an urgent care clinic with a history of low-grade fevers, diffuse abdominal pain, and multiple episodes of non-bloody diarrhea for the last two days. He also had one to two episodes of non-bloody, non-bilious emesis each day, but this is slowing down. He is not eating as well as usual, but he is taking small bites and drinking plenty of fluids. On evaluation, he appears tired and is not playful, but his vital signs are normal and he appears to be well-hydrated. Which of the following is the most appropriate therapeutic approach for this patient? A) Antiemetic medication B) Antimotility medication C) Probiotic treatment D) Zinc supplementation
C) Probiotic treatment (most likely viral so probiotic will replenish normal intestinal flora as the disease self-limits)
A five-year-old boy presents to the clinic with his parents, who state he had a syncopal episode which lasted less than one minute and spontaneously resolved. The syncope happened while the child was running in the house. An ECG is performed and shows QT segment duration of 500 milliseconds. Family history is positive for a paternal grandfather who died suddenly in his fifties of unspecified cardiac problems. Which of the following represents the best initial therapeutic choice for this patient? A) Implantable cardioverter-defibrillator B) Metaproterenol 0.4 mg/kg every 12 hours C) Propranolol 2.9 mg/kg/day D) Radiofrequency ablation of accessory pathways
C) Propranolol 2.9 mg/kg/day (B-blockers are first line)
Which of the following descriptions is more characteristic of urticaria than any other type of skin abnormality? A) Grouped vesicles with underlying erythema B) Papules with burrows C) Pruritic erythematous plaques D) Tender fluctuant nodules
C) Pruritic erythematous plaques
A five-year-old boy has a height which is 2.1 standard deviations below normal for his age. His height velocity is six centimeters per year. When calculated, the boy's projected height is 2.2 standard deviations below the mid-parental height. His parents state he has been in good health and no abnormalities are found on physical exam other than short stature. Which of the following is the most appropriate next step in evaluation of this boy's short stature? A) Complete blood count and erythrocyte sedimentation rate to screen for anemia or autoimmune disease B) Karyotype testing to screen for Turner syndrome C) Radiographs of left wrist and hand to determine bone age D) Tissue transglutaminase IgA test to screen for celiac disease
C) Radiographs of left wrist and hand to determine bone age
A 10-year-old boy with a known history of environmental allergies and diagnosis of mild intermittent asthma presents for a routine visit and medication refill. He is currently managed on an albuterol inhaler as needed. Which of the following, if reported by the family, would indicate a need to advance his management to include a daily low-dose inhaled glucocorticoid? A) Asthma symptoms during cold-weather months B) Asthma symptoms three days per month C) Asthma symptoms three nights per month D) Asthma symptoms when exercising
C) Asthma symptoms three nights per month
A 2-year-old boy, who is up to date with his vaccinations, presents in mild respiratory distress with noticeable inspiratory stridor and a barking cough on exam. His parents report his symptoms seemed to improve with exposure to the cold outside air. What is the most likely pathogen responsible for the patient's presentation? A) Bordetella pertussis B) Haemophilus influenzae type B C) Parainfluenza virus D) Respiratory syncytial virus
C) Parainfluenza virus (MC cause of croup)
A 12-year-old boy presents with three days of fever, malaise, and sore throat. Which favors a diagnosis of infectious mononucleosis over streptococcal pharyngitis? A) Fever B) Pharyngeal exudates C) Posterior cervical adenopathy D) Tender bilateral anterior cervical adenopathy
C) Posterior cervical adenopathy -anterior favors strep pharyngitis -A and B are signs of both
When does surfactant begin to be expressed in the fetal lung? A) 12 weeks gestation B) 20 weeks gestation C) 28 weeks gestation D) 35 weeks gestation
B) 20 weeks gestation (complete by 33-36 weeks)
Normally, when lactose is taken into the digestive system, it is broken down by the enzyme lactase. What two products are created by this process? A) Fructose and galactose B) Fructose and glucose C) Galactose and sucrose D) Glucose and galactose
D) Glucose and galactose
It is postulated that antibody-antigen complexes are responsible for some viral exanthems because the rash does not appear until after the patient begins to recover. Which of the following viral illnesses follows this pattern of rash appearance concurrent with recovery? A) Hand, foot, and mouth disease B) Kawasaki disease C) Rocky Mountain spotted fever D) Roseola
D) Roseola
An 11-month-old girl presents to the pediatrician with her mother, who states that the child is has not spoken any words since birth. The mother is concerned because the girl has even stopped making the noises she used to make. She does not seem to respond to voices or other noises in the room. On physical exam the external auditory canal appears patent. The tympanic membrane can be visualized and no abnormalities are noted. Which of the following would be appropriate to include in the physical exam at this time? A) Ophthalmology exam B) Rinne test C) Speech audiometry D) Weber test
A) Ophthalmology exam *all other tests are not useful in infants*
A 28-month-old boy presents with two days of fever and cough. Over the past several hours, he has developed subcostal retractions. In triage, room air oxygen saturations are 86% and the respiratory rate is 36 breaths/minute. He has crackles in his left lower lobe on auscultation. What is the most likely diagnosis? A) Asthma exacerbation B) Bronchiolitis C) Community-acquired pneumonia D) Foreign body aspiration
C) Community-acquired pneumonia
A 6-week-old girl presents to clinic with whitish plaques on her tongue, palate, and inner cheeks. Her parents have attempted to remove it, but it led to bleeding. The duration of each breastfeeding session has decreased from 20 minutes to 10 minutes as the plaques have worsened. Her parents are concerned that she may have streptococcal pharyngitis, as her older sister was recently diagnosed. What is the most likely diagnosis? A) Malignancy B) Milk on tongue C) Oropharyngeal candidiasis D) Streptococcal infection
C) Oropharyngeal candidiasis
A 4-year-old girl presents with left ear pain and fever for four days. There is no tenderness with manipulation of the tragus, but there is pain of the bony surface posterior to the ear. The left ear appears anteverted. Visualization of tympanic membrane is limited, but profuse clear-white discharge is present within the external auditory canal. What is the most likely diagnosis? A) Lymphadenitis B) Mastoiditis C) Otitis externa D) Retropharyngeal abscess
B) Mastoiditis (complication of otitis media) -suspected with tenderness of the postauricular area
Which of the following findings would be expected on physical exam of a child presenting to the clinic with suspected mumps? A) Koplik spots B) Parotitis C) Splenomegaly D) Strawberry tongue
B) Parotitis
A 2-year-old girl presents to the emergency room because of rectal bleeding. The mother noticed the reddish-colored stools three days ago and has since changed several diapers with just blood. Past medical and family history is unremarkable. The child is afebrile, alert and active, but slightly tachycardic. Other than finding a small amount of blood in the diaper, the rest of the physical examination is normal. What would be the best diagnostic step for confirmation of diagnosis? A) Plain abdominal radiography B) Stool culture C) Technetium scan D) Whole abdominal ultrasound
C) Technetium scan (DOC for Meckel's diverticulum)
A 15-year-old previously healthy boy presents with a complaint of his right eye being stuck shut in the morning, with discharge from the eye continuing throughout the day. On physical exam, the conjunctiva is erythematous with a copious purulent, green discharge. Which of the following is the most likely diagnosis? A) Allergic conjunctivitis B) Bacterial conjunctivitis C) Nonallergic, noninfectious conjunctivitis D) Viral conjunctivitis
B) Bacterial conjunctivitis
A 14-year-old boy presents with two erythematous plaques on his right arm. The plaques are pruritic and have developed a mild scale and central clearing. Other boys on the patient's wrestling team have similar lesions. Which of the following is most likely to confirm the suspected diagnosis? A) Herpes simplex virus polymerase chain reaction of unroofed lesion B) Potassium hydroxide preparation C) Punch biopsy D) Response to topical steroids
B) Potassium hydroxide preparation (DOC for tinea corporis)
A 3-year-old girl is brought in for her regular well-child exam and a cardiac murmur is auscultated on physical exam. The short, systolic murmur is grade II in intensity, softer in intensity when she is sitting, and has a "musical" quality. What is the most likely diagnosis? A) Atrial septal defect B) Innocent murmur C) Pericarditis D) Ventricular septal defect
B) Innocent murmur -short -systolic -less intense while sitting or upright -more intense when supine
A 5-year-old previously healthy girl presents with right ear pain and discharge for 2 days. She has not had fever, nasal discharge, or cough. On examination, her right ear is normally placed. She has tenderness with movement of the tragus and extreme tenderness with examination of the external auditory canal. The canal is diffusely erythematous with scant pus present. The visualized portions of the tympanic membrane appear normal. What is the most likely diagnosis? A) Acute otitis media B) Fungal otitis externa C) Malignant otitis externa D) Otitis externa
D) Otitis externa MCC: pseudomonas aeruginosa, staph epidermis, and staph aureus TOC: topical antibiotics, avoid topical ahminoglycosides if TM cannot be visualized
A 4-month-old infant presents with three days of cough and nasal congestion. Today, the parents note that he has been breathing faster and harder. On examination, he is tachypneic with moderate subcostal retractions and nasal flaring. He has diffuse crackles and scattered expiratory wheezes. What is the most likely diagnosis? A) Bronchiolitis B) Community-acquired pneumonia C) Foreign body aspiration D) Status asthmaticus
A) Bronchiolitis
A healthy, full-term 2-month-old girl presents to her pediatrician for follow-up. She has been gaining weight as expected and has not had any illnesses other than mild cold symptoms. At her last routine well-child exam, a one-out-of-six mid-systolic pulmonary flow murmur was appreciated, and she was sent for an echocardiogram. The results of the echocardiogram were notable for an isolated secundum 2.5 mm atrial septal defect. After explaining the nature of this finding, her parents are very concerned and have many questions. What is your recommendation for follow-up? A) No special follow-up is needed unless new signs or symptoms develop B) Referral to a pediatric cardiac surgeon for percutaneous closure C) Referral to a pediatric cardiologist for close follow-up D_ Start high-calorie formula with close growth monitoring
A) No special follow-up is needed unless new signs or symptoms develop For ASD: -typically isolated defect -common -most heal on their own
A 6-week-old baby girl presents to the pediatrician for her checkup. The mother states that her daughter seems healthy and is eating well. Her growth chart shows an upward trend. There is no evidence of jaundice. Her labs show a hemoglobin of 11 g/dL, and 1 month ago, her hemoglobin was 14 g/dL. What is the most likely cause of the decreased hemoglobin level? A) Hemoglobinopathy B) Hemolytic anemia C) Iron deficiency anemia D) Physiologic anemia
D) Physiologic anemia (MC cause of anemia in young infants 6-9 weeks of age)
A 14-year-old boy presents with one month of increasing knee pain. Initially, he noted the pain in his right knee, but now his left knee is also bothering him. His parents note he has been walking with a limp, which at first improved when treated with ibuprofen but now he seems to be getting worse. He denies recent injury or illness, fever, swelling or point tenderness in either lower extremity. He is not having night symptoms, and he is not experiencing numbness or tingling. The pain is most noticeable with weight-bearing activities. On initial inspection you note an obese adolescent boy, with limping gait, and no apparent lower extremity irregularities. During your exam, no tenderness to palpation over the lower extremity is found, including the knee joints bilaterally, but manipulation is significant for decreased range of motion of the hips bilaterally, particularly internal rotation and abduction. Which of the following is the most likely diagnosis? A) Juvenile idiopathic arthritis B) Legg-Calvé-Perthes disease C) Osgood-Schlatter disease D) Slipped capital femoral epiphysis
D) Slipped capital femoral epiphysis
A 7-year-old boy presents to the primary care physician with his parents. The parents state that he would not eat this morning and was complaining that it hurt to swallow and to urinate. He has been running a fever for the past week and is up to date on his vaccinations. He has a past medical history of epilepsy and started a new medication, carbamazepine, two weeks ago for seizures. On physical exam there are intraoral hemorrhagic erosions covered with a greyish white membrane on the oral mucosa. There are also lesions surrounding his vermillion border. He has a severe case of conjunctivitis. Which of the following is the most likely diagnosis? A) Diphtheria B) Herpetic gingivostomatitis C) Staphylococcal scalded skin syndrome D) Stevens-Johnson syndrome
D) Stevens-Johnson syndrome -commonly caused by Carbamazepine